Laboratory Dha Praparation MCQ 1-2700
Laboratory Dha Praparation MCQ 1-2700
Laboratory Dha Praparation MCQ 1-2700
a. Epithelial Tissue
b. Muscle tissue
c. Connective Tissue ✔
d. Nervous Tissue
02. Mycoplasmas are differentfrom other bacteria by?
a. Presence of chitin in cell wall
b. Presence of Techoic acid in cell wall
c. Presence of Lipoprotein in cell wall
d. Absence of cellwall itself. ✔
03. The Difference of Plasma and Serum is that Plasma ?
a. Dosenot contain fibrinogen.
b. has more water
c. Contain Fibrinogen. ✔
d. Contain Iron.
04. New Methylene Blue Reagent is used for staining of which blood cells?
a. Reticulocytes. ✔
b. Platelets.
c. WBC’s
d. Heinz bodies
05. Gram Staining was introduce by which scientist ?
a. Alfred Gram
b. Christian Gram. ✔
c. Robert Cook
d. Leuis Pasteur
06. When WBC are counted manually by tuk’s Reagent , Blood is diluted by Ratio ?
a. 1:20 ✔
b. 1:50
c. 1:100
d. 1:200
07. The end Point of Purine Metabolism is ?
a. Urea
b. Creatinin
c. Uric Acid ✔
d. Ammonia
08. Which is the following chemical test differentiate between Staphylococcus and Streptococcus ?
a. Coagulase Test
b. Catalase Test ✔
c. Urase Test
d. Oxidase Test
09. Which is the following Immunoglobulin can cross Placenta ?
a. IgA
b. IgD
c. IgG ✔
d. IgM
10. All of the following are function of blood except one ?
a. Hormone Production ✔
b. Buffer system
c. Oxygen Transport
d. Nutrient Absorption
11. Which is the following test is used to detect/determine Syphilis ?
a. ASOT
b. Ra factor
c. CRP
d. VDRL ✔
12. One of the following is major cation found in extracellular fluid ?
a. Sodium ✔
b. Potassium
c. Magnesium
d. Zinc
13. Immunoglobuline are which type of Protein in Nature ?
a. Alpha Globulin
b. Beta Globulin
c. Gamma Globulin ✔
d. Delta Globulin
14. Entrobius Vermicularis is also known as ?
a. Pin worm ✔
b. Round worm
c. Thread Worm
d. Whip Worm
15. Which is the following hormone directly Regulate Blood Glucose Level ?
a. TSH
b. FSH
c. Progesterone
d. Insulin ✔
16. Which is the following Anticoagulant Tube need to collect sample of HbA1c ?
a. Sodium Flouride
b. EDTA ✔
c. Heparin
d. Sodium Sitrate
17. Which is the following Parasite Doesn’t develop Systic stage ?
a. Giardia Lambia
b. Antamoaba Histolytica
c. Enterobius Vermicularis
d. Trichomonas Vaginalis ✔
18. A Blood Donor who recently diagnoses Positive HbsAg should be deffered for how long ?
a. Permanantly ✔
b. For 3 months
c. For 6 Months
d. For 1 Year
19. Which is the following is called “bad” Cholestrol ?
a. Cholestrole
b. Triglyceride
c. HDL
d. LDL ✔
20. Which one of the following test is used to differentiate Staphylococcus aureus from other Staphylococcus
Species ?
a. Coagulase Test ✔
b. Catalase Test
c. Urase Test
d. Oxidase Test
21. Hemophilia A is caused by which factor deficiency ?
a. Factor VIII ✔
b. Factor IX
c. Factor X
d. Factor I
22. Salmonella and Shigella can be differentiate on the base of which test ?
a. Gram Stain
b. Catalase Test
c. Motility ✔
d. Indole Test
23. Ammonium Oxalate reagent is used for the counting of which test ?
a. WBC’s
b. Platelets ✔
c. RBC’s
d. Reticulocytes
24. Which of the following Bio-chemistry Test Should keep away from light ?
a. Urea
b. Bilirubin ✔
c. Glucose
d. calcium
25. Which one of the following Immunoglobulin appeared as Pantamer ?
a. IgA
b. IgE
c. IgG
d. IgM ✔
26. The solution used to fix a pap smear is:
a. wright stain
b. hematoxylin
c. physiological saline
d. cytospray ✔
27. Which on of the following bacteria can cause glomarular nephritis or FR ?
a. Staphylococcus aureus
b. Streptococcus Pyogenes ✔
c. Staphylococcus Epidermidis
d. Streptococcus Pneumonia
28. Which one of the following hormone is produced by pituitary gland ?
a. TRH
b. T4
c. TSH ✔
d. Insulin
29. in DIC all of the following test will show high results except ?
a. PT
b. Platelets ✔
c. APTT
d. None of the above
30. Three types of ketone bodies include acetoacetic acid, acetone and __?
a. Oxaloacietic acid
b. Lactic Acid
c. 3-hydroxy Butilic acid ✔
d. Acietic Acid
31. The degree that a control results deviate from known valus or mean value is known as :-
a. Standard Deviation ✔
b. Coefficient Deviation
c. Median
d. None of above
32. Erythroblastosis Foetalis accurs due to :-
a. ABO Incompatibility
b. RH Incompatibility ✔
c. Hemophilia
d. Leukemia
33. One of the following enzyme also performed cardiac enzyme function ?
a. Acid Phosphatase
b. Creatinine
c. LDH ✔
d. ALT
34. ESR will show low value result in which one of the following condition :-
a. Polycythemia ✔
b. Anemia
c. RA
d. Tuberculosis
35. Which one of the following paracite causing ameobic dysentery ?
a. Giardia Lambia
b. Entamoeba Histolytica ✔
c. Ascaris Lumbericodes
d. Shigella
36. Megaloblastic anaemia is caused by deficiency of folate and which vitamin ?
a. Vitamin A
b. Vitamin B12 ✔
c. Vitamin D
d. Vitamin C
37. Which One of the Following bacteria is gram negative, Aerobic oxidase positive and motile rod ?
a. E- Coli
b. Proteus
c. Salmonella
d. Pseudomonas ✔
38. Which one of the following cells play a major role in humoral immunity ?
a. B – Cells ✔
b. T – Cells
c. Monocytes
d. Macrophages
39. In Iron Deficiency anemia laboratory finding which one of the following will show increased value ?
a. Hb
b. Iron
c. TIBC ✔
d. RBC’s
40. Widal test is done for the diagnosis of which bacteria ?
a. Shigela
b. E- Coli
c. Salminella ✔
d. Proteus
41. If Sample is taken in EDTA Tube, Which one of the following biochemistry test results will affect ?
a. Glucose
b. Urea
c. Cholestrol
d. Calcium ✔
42. Ziel Nelson stain is used for ?
a. Streptococcus
b. Mycobacterium ✔
c. H Influenza
d. Bacillus sp
43. Co-efficient of variation is calculated by which formula ?
a. Sd x 100 / Standard Error (SD=Standard Deviation)
b. Mean x 100 / SD
c. SD x 100 / Mean ✔
d. Variance x 100 / Mean
44. Night Blindness is caused by deficiency of which vitamin ?
a. Vitamin C
b. Vitamin A ✔
c. Vitamin B12
d. Vitamin B6
45. What is erythropoietin ?
a. is secreted by kidney
b. Stimulates the bone-marrow to produce RBC’s
c. Is released in responce to hypoxemia
d. All of above ✔
46. Each of following characteristics is correct about Ascaris lumbricoides except one :-
a. Ascaris lumbricaid is nematode
b. Both dogs and cats are intermediate host of Ascaris lumbricoides ✔
c. Ascaris lumbricoides can cause pneumonia
d. Ascaris lumbricoides is transmetted by enjections of eggs
47. Which types of cells if rapidly can cause jaundice?
a. WBC’s
b. Platelets
c. Plasma Cells
d. Red Blood Cells ✔
48. Kala- Azar Disease is caused by which Parasite ?
a. Lieshmania donovani ✔
b. Trypanosoma Cruzi
c. Wucheria Bancrofti
d. Plasmodium
49. When myocardial infarction will happen which of the following cardiac enzyme will elevate first ?
a. LDH
b. CK ✔
c. ALT
d. AST
50. Cason’s test is used for the diagnosis of which parasite ?
a. Taenasis
b. Hydatidasis ✔
c. Trichurosis
d. Onchuriasis
51. In case of acute hepatitis which liver enzyme is more Specific to check ?
a. ALT ✔
b. AST
c. ALP
d. Both ALT and AST
52. The conversion of glucose into lactate or Pyruate is called ?
a. Glycolysis ✔
b. Glycopenesis
c. Glycogenolysis
d. Gluconeogensis
53. Red Cells which are to be tested antiglobulin reagent are washed to :-
a. Remove traces of bacterial protein.
b. Wash away traces of free haemoglobin.
c. Remove Unbound serum globulin ✔
d. expose additional antigen site.
54. The failure of granulocytes (Neutrophils) to develop band or two lobed stage is characteristic of |:-
a. Bernard-Soulier syndrome (BSS)
b. Chédiak–Higashi syndrome (CHS)
c. May-Hegglin anomaly (MHA)
d. Pelger-huet anomaly (PHA) ✔
55. An elevated urine urobilinogen and negative test for urine bilirubin may indicate which of following
condition ?
a. Acute Hepatic Toxicity
b. Biliary Abstruction
c. Hemolytic Diseases ✔
d. Urinary Tract Infection (UTI)
56. The most common light source for spectrophotometery in the visivble range id :-
a. Didymium Lamp
b. Duterium Lamp
c. Hydrogen discharge lamp
d. Tungusten Lamp ✔
57. A ‘rice water stool’ is characteristics of patient infected with :-
a. Clostridium botulinum
b. Salmonella Typhi
c. Shigella dysenteriae
d. Vibrio Cholera ✔
58. The Major Iron storage compound is :-
a. Hemosodium
b. Ferritin ✔
c. Siderotic Granules
d. Transferrin
59. The protein fraction that migrates most rapidly toward anode is :-
a. Albumin ✔
b. Beta
c. Gamma
d. Alpha 1
60. Leoffler’s medium is used as a primary isolation medium for :-
a. Bordetella Pertusis
b. Corynebacterium diphtheriae ✔
c. Clostridium tetani
d. Streptococcus Pyogenes
63. Which Trophozoite is suspected if ingested red cells are seen on a saline wet preparation ?
a. Entamoeba Coli
b. Entamoeba histolytica ✔
c. Endolimax nana
d. Trichomonas Vaginals
64. Myltiple Myeloma may be suspected which one of the following is seen on a Peripheral Smear:-
a. Basophilic stippling
b. Blast Cells
c. Hypersegmented neutrophils
d. Rouleaux ✔
65. Which one of the following test is used to monitor a diabetic patient long term control ?
a. Glucose Tolerance Test
b. Glycosylated Haemoglobin ✔
c. Fasting Blood Sugar
d. Postprandial blood sugar
66. Which one of the following result is typical of compylobacter jejuni ?
a. Catalase Negative
b. Non Motile
c. Optimal Growth at 42°C ✔
d. Oxidase Negative
67. Which one of the following leukocyte count will increase in parasitic and allergic reactions ?
a. Neutrophils
b. Lymphocytes
c. Eosinophils ✔
d. Basophils
68. Enterobius vermicularis infaction is usually diagnosed by finding egg from:-
a. Cellulose tap prep ✔
b. Concentrated Stool Sample
c. Iodine wet mount
d. Sedimented Stool sample
69. Plasma cells evolve is which cell line ?
a. Lymphocytic ✔
b. Monocytic
c. Myelocytic
d. Megamyelocytic
70. The Biuret method of determing serum total protein is dependent upon :-
a. Amino acid content
b. number of peptide bond ✔
c. Nitrogen content
d. Protein precipitation
71. Which Characteristic is most useful in differentiating citrobacter and salmonella :-
a. H2s Production
b. Indole Production
c. Lysine decarboxylase ✔
d. Urase Production
72. Pregnant women are to avoid cleaning litter boxes of there house cats until after delivery to prevent
congenital infaction for :-
a. Ancylostoma caninum
b. Giardia Lambia
c. Diphyllobothrium latium
d. Toxoplasma gondii ✔
73. Prolonged bleeding time and giant platelets best describes :-
a. Bernard Soulier Syndrome (BSS) ✔
b. Glanzmann’s thrombasthenia
c. Von Willebrand disease (VWD)
d. Wiskott–Aldrich syndrome (WAS)
74. A Patient in diabetic ketoacidosis (DKA) would exhibit a:-
a. ↑ pCO2 , ↑HCO3 , ↑PH
b. ↓pCO2 , ↓ HCO3 , ↓PH
c. Normal pCO2 , ↑HCO3 , ↓PH
d. Normal pCO2 , ↓ HCO3 , ↓PH ✔
75. Another name of natural immunity of :-
a. Innate Immunity ✔
b. Acquired Immunity
c. Adoptive Immunity
d. Cellular Immunity
76. Which one of the following is used as urine Preservative :-
a. Formaldehyde
b. Boric Acid ✔
c. Gluteraldehyde
d. Picric Acid
77. Hydrops Fetalis mean :-
a. One deleted α-Genes (-α/αα)
b. Two deleted α-Genes (- -/αα)
c. Three deleted α-Genes (- -/- α)
d. Four deleted α-Genes (- -/- -) ✔
78. Value of the control that continue to ether increase or decrease over a period of 6 consecutive days is called
:-
a. Shift
b. Trend ✔
c. Standard Deviation
d. Co-efficient of variation
79. X-Factor and V-Factor is the characteristics of which bacteria :-
a. Hemophilus ✔
b. Brucella
c. Legionella
d. Salmonella
80. Which of the following would cause an individual to rejected as blood donor :-
a. Plus of 95
b. 13 g/dl haemoglobin
c. Age 50
d. Blood pressure 180/120 mm of mercury ✔
81. Which of the following results from a rate of synthesis abnormally ?
a. Bsta Thalassemia
b. Haemoglobin CC Disease
c. Haemoglobin Lepore Syndrome
d. Sickle Cell Disease ✔
82. Which of the following is an immune-mediated condition characterised by low platelet count and found
primarily in children ?
a. Idiopathic thrombocytopenic purpura
b. Thrombotic thrombocytopenic purpura
c. May hegglin ✔
d. Wiskott–Aldrich
83. If there is an increase in abnormal volume of urine, the condition is called as :-
a. Polyuria ✔
b. Oliguria
c. Anuria
d. Nocturia
84. Sodium Nitroprusside present on the dipstick used for the measurement of :-
a. Glucose
b. Protein
c. Ketone ✔
d. Urobilinogen
85. Measures light that is scattered by small particles at right angle to the beam incident of the cuvette in the
principle of :-
a. Fluorometry
b. Turbidimetry
c. Nephelometry ✔
d. Osmomatery
86. Method of the physical separation of proteins based of there Ionic charge and molecular size is principle of
:-
a. Spectrophotometer
b. Electrophoresis ✔
c. Fluorometry
d. Osmomatery
87. Alpha cells of pancreas Producing :-
a. Glucogen ✔
b. Insuline
c. Cortisol
d. Epinephrine
88. Which of the following disease will show increase level of alkaline Phosphatase :-
a. Hepatitis
b. 3rd Trimester of Pregnancy
c. Obstactive Juindice
d. Both “2” and “3” ✔
89. Which one of the following level is maintained in body by Parathyroid gland :-
a. Sodium
b. Potassium
c. Magnessium
d. Calcium ✔
90. Direct bilirubin is also called as :-
a. Unconjugated bilirubin
b. Conjugated Bilirubin
c. Water soluble bilirubin
d. Both “2” and “3” ✔
91. Increase Cortisol level is seen in which syndrome ?
a. Addison’s disease
b. Cushing Syndrome ✔
92. Different between N. Meningitis and N. Gonorrhea is that N. Meningitis ferment only :-
a. Glucose
b. Maltose ✔
c. Sucrose
d. Lactose
93. Chinese letter arrangement, Metachromatic granules, tellurite hydrolysis and elek test is characteristic of
which bacteria ?
a. Listeria monocytogenes
b. Bacillus anthracis
c. Corynebacterium diphtheriae ✔
d. Moraxella catarrhalis
94. Which of the following bacteria is urase positive, Indole Positive, and H2S Positive ?
a. Proteus ✔
b. Escherchia
c. Salmonella
d. Shigella
95. Seagull appears is characteristic of which bacteria ?
a. Campylobacter Jejuni ✔
b. Vibrio cholerae
c. E- Coli
d. Salmonella
96. Which of the following bacteria Stain which silver impregnation and can be seen by Dark field microscopy
?
a. Treponema pallidum ✔
b. Homeopathic influenza
c. Streptococcus pyogenes
d. Spirillum minus
97. Lowenstein-Jensen (LJ) is used for the growth of which of the following :-
a. Staphylococcus
b. Streptocossus
c. Neisseria
d. Mycobacteria ✔
98. A cause of acute factitious infantile diarrhea is :-
a. hantavirus
b. HIV
c. Rhabdovirus
d. Rotavirus ✔
99. Specimen for viral culture should be transported in :-
a. Anaerobic Container
b. Bovine Albumin
c. Nutrient medium with antibiotic ✔
d. Sheep blood (5 – 10 %)
100. In order to prove a yeast is dimorphic, which of the following test is performed ?
a. Carbohydrate assimilation
b. Growth on Corn Meal Agar,
c. Incubate yeast subculture at 37°C ✔
d. Urease
101. The infrective stage for strongyloides stercoralis is :-
a. Ova
b. Filariform larvae ✔
c. Rhabditiform Larva
d. Free living adult
102. If human ingest the egg of taenia Solium , they may devolp :-
a. Hydatid Disease
b. Sparganosis
c. Trichinosis
d. Cysticercosis ✔
103. Which of the following egg may not be detected in zinc flotation procedure ?
a. Ascaris Lumbricoides
b. Trichua Trichuris
c. Schistosoma mansoni ✔
d. Taenia solium
104. Which organisms is transmitted to humans by ticks and blood transfusion ?
a. Babesia ✔
b. Rickettsia rickettsii
c. Leishmania donovani
d. Trypanosoma Cruzi
105. Which of the following is most specific test for syphilis ?
a. RPR
b. TPHA
c. FTA-Abs ✔
d. VDRL
106. Which of the following used in mantoux test ?
a. Lipoprotein
b. Carbohydrate
c. Purified Protein Derivatives
d. Hisitidine-rich protein II ✔
107. Gram Positive bacteria differ from Gram Negative by which of the following ?
a. Peptidoglycan (murein)
b. Mycolic acid
c. Teichoic acid ✔
d. All of above
108. What is the end product of purine metabolism ?
a. Urea
b. Xanthine
c. Uric Acid ✔
d. Guanine
109. Which Stain used commonly in psychopathology ?
a. H & E
b. Alcain Blue
c. Giemsa Stain
d. Pap Stain ✔
110. Largest virus is :-
a. Parvo Virus
b. Herpes Virus
c. Pox Virus ✔
d. Mycoplasma
111. How many pairs of Sex chromosome Present in Human ?
a. 22
b. 46
c. 23
d. 01 ✔
112. Pork Tapeworm is :-
a. Taenia Solium ✔
b. Taenia saginata
c. Enterobius Vermicularis
d. Plasmodium
a. Alpha-1 globulin
b. Alpha1- globulin and alpha-2 globulin
c. Alpha-2 globulin and beta globulin ✔
d. Beta globulin and gamma globulin
1105. The following lab results were obtained from arterial blood: sodium 136 mEq/L pH 7.32
Potassium 4.4 mEq/L PCO2 79 mm Hg Chloride 92 mEq/L Bicarbonate 40 mEq/L these results
are compatible with?
a. Respiratory alkalosis
b. Respiratory acidosis ✔
c. Metabolic alkalosis
d. Metabolic acidosis
1106. Which of the following statements about fluorometry are true?
a. A compound is said to fluoresce when it absorbs light at one wavelength and emits light at a second wave
length ✔
b. Detectors in fluorometers are placed 180 degrees from the excitation source
c. It is less sensitive than spectrophotometry
d. It avoids the necessity for complexing of components because fluorescence is a native property
1107. Two standard deviations is the acceptable limit of error in the chemistry laboratory. If you run the normal control
100 times, how many of the values wou;d be out of control due to random error?
a. 1
b. 5 ✔
c. 10
d. 20
1108. Serum anion gap is increased in patients with?
a. Renal tubular acidosis
b. Diabetic alkalosis
c. Metabolic acidosis due to diarrhea
d. Lactic acidosis ✔
1109. The nanometer is used as a measure of?
a. Absorbance
b. % transmittance
c. Intensity of radiant energy
d. Wavelength of radiant energy ✔
1110. A common cause of respiratory alkalosis is?
a. Vomiting
b. Starvation
c. Asthma
d. Hyperventilation ✔
1111. Major actions of angiotensin II include?
a. Increased pituitary secretion of rennin
b. Increased vasoconstriction ✔
c. Increased parathyroid hormone secretion by the parathyroid
d. Decreased adrenal secretion of aldosterone
1112. The anion gap is useful for QC of laboratory results for?
a. Amino acids and proteins
b. Blood gas analyses
c. Sodium, potassium, chloride, and total CO2 ✔
d. Calcium, phosphorus, and magnesium
1113. Which of the following electrolytes is the chief plasma cation whose main function is maintaining osmotic
pressure?
a. Chloride
b. Calcium
c. Potassium
d. Sodium ✔
1114. What is the molarity of a solution that contains 18.7 g of KCI (MW=74.5) in 500 ml of water?
a. 0.1
b. 0.5 ✔
c. 1.0
d. 5.0
1115. The buffering capacity of blood is maintained by a reversible exchange process between bicarbonate and?
a. Sodium
b. Potassium
c. Calcium
d. Chloride ✔
1116. The different water content of erythrocytes and plasma makes true glucose concentrations in whole blood a
function of the?
a. Hematocrit ✔
b. Leukocyte count
c. Erythrocyte count
d. Erythrocyte indices
1117. Acidosis and alkalosis are best defined as fluctuations in blood pH and CO2 content due to changes in?
a. Bohr’s effect
b. O2 content
c. Bicarbonate buffer ✔
d. Carbonic anhydrase
1118. Metabolic acidosis is described as a(n)?
a. Increase in CO2 content and PCO2 with a decreased pH
b. Decrease in CO2 content with an increased pH
c. Increase in CO2 with and increased pH
d. Decrease in CO2 content and PCO2 with a decreased pH ✔
1119. The normal range for the pH of arterial blood measured at 37 degrees Celsius is?
a. 7.28-7.34
b. 7.33-7.37
c. 7.35-7.45 ✔
d. 7.45-7.50
1120. Hydrogen ion concentration (pH) in blood is usually determined by means of which of the following electrodes?
a. Silver
b. Glass ✔
c. Platinum
d. Platinum-lactate
1121. The conversion of glucose or other hexoses into lactate or pyruvate is called?
a. Glycogenesis
b. Glycogenolysis
c. Gluconeogenesis
d. Glycolysis ✔
1122. A reciprocal relationship exists between?
a. Sodium and potassium
b. Calcium and phosphorus ✔
c. Chloride and CO2
d. Calcium and magnesium
1123. Respiratory acidosis is described as a(n)?
a. Increase in CO2 content and PCO2 with a decreased pH ✔
b. Decrease in CO2 content with an increased pH
c. Increase in CO2 content with an increased pH
d. Decrease in CO2 content and PCO2 with a decreased pH
1124. Which of the following hemoglobins has glucose-6-phosphate on the amino-terminal valine of the beta chain?
a. S
b. C
c. A2
d. A1C ✔
1125. The function of the major lipid components of the very low density lipoproteins (VLDL) is to transport?
a. Cholesterol from peripheral cells to the liver
b. Cholesterol and Phospholipids to peripheral cells
c. Exogenous triglycerides
d. Endogenous triglycerides ✔
1126. A hospitalized patient is experiencing increased neuromuscular irritability (tetany). Which of the following tests
should be ordered immediately?
a. Calcium ✔
b. Phosphorus
c. BUN
d. Glucose
1127. Sixty to seventy-five percent of the plasma cholesterol is transported by?
a. Chylomicrons
b. VLDL
c. LDL ✔
d. HDL
1128. Measurement of total T4 by competitive protein binding or displacement is based on the specific binding
properties of?
a. Thyroxine-binding prealbumin
b. Albumin
c. Thyroxine-binding globulin ✔
d. Thyroid-stimulating hormone
1129. Which of the following methods employs a highly specific antibody to thyroxine?
a. Total T4 by competitive protein binding
b. T4 by RIA ✔
c. T4 by column
d. T4 by equilibrium dialysis
1130. The extent to which measurements agree with the true value of the quantity being measured is known as?
a. Reliability
b. Accuracy ✔
c. Reproducibility
d. Precision
1131. When myocardial infarction occurs, the first enzyme to become elevated is?
a. CK ✔
b. LD
c. AST
d. ALT
1132. In the Jaffe reaction, creatinine reacts with?
a. Alkaline sulfasalazine solution to produce an orange-yellow complex
b. Potassium iodide to form a reddish-purple complex
c. Sodium nitroferricyanide to yield a reddish-brown color
d. Alkaline picrate solution to yield and orange-red complex ✔
1133. Which of the following represents the end product of purine metabolism in humans?
a. AMP and GMP
b. DNA and RNA
c. Allantoin
d. Uric acid ✔
1134. The biological most active, naturally occurring androgen is?
a. Androstenedione
b. Dehydroepiandrosterone
c. Epiandrosterone
d. Testosterone ✔
1135. Bile acid concentrations are useful to asses?
a. Diabetes mellitus
b. Hepatobiliary disease ✔
c. Intestinal Malabsorption
d. Kidney function
1136. Which of the following is secreted by the placenta and used for the early detection of pregnancy?
a. Follicle-stimulating hormone (FSH)
b. Human Chorionic gonadotropin (HCG) ✔
c. Luteinizing hormone (LH)
d. Progesterone
1137. Decreased serum iron associated with increased TIBC is compatible with which of the following disease states?
a. Anemia of chronic infection
b. Iron deficiency anemia ✔
c. Chronic liver disease
d. Nephrosis
1138. The principle excretory form of nitrogen is?
a. Amino acids
b. Creatinine
c. Urea ✔
d. Uric acid
1139. The parent substance in the biosynthesis of androgens and estrogens is:
a. Cortisol
b. Catecholamines
c. Progesterone
d. Cholesterol ✔
1140. In the Malloy and Evelyn method for the determination of bilirubin, the reagent that is reacted with bilirubin to
form a purple azobilirubin is:
a. Dilute sulfuric acid
b. Diazonium sulfate
c. Sulfobromophthalein
d. Diazotized sulfanilic acid ✔
1141. Maple syrup urine disease is characterized by an increase in which of the following urinary amino acids?
a. Phenylalanine
b. Tyrosine
c. Valine, leucine, and isoleucine
d. Cystine and cysteine ✔
1142. Which of the following enzymes catalyzes the conversion of starch to glucose and maltose?
a. Malate dehydrogenase (MD)
b. Amylase (AMS) ✔
c. Creatine kinase (CK)
d. Isocitric dehydrogenase (ICD)
1143. The greatest activities of serum AST and ALT are seen in?
a. Acute hepatitis ✔
b. Primary Biliary cirrhosis
c. Metastatic hepatic carcinoma
d. Alcoholic cirrhosis
1144. Bioavailability of a drug refers to the?
a. Availability for therapeutic administration
b. Availability of the protein-bound fraction of the drug
c. Drug transformation
d. The fraction of the drug absorbed into the systemic circulation ✔
1145. The cyclic antidepressants are classified as?
a. Basic drugs ✔
b. Neutral drugs
c. Acidic Drugs
d. Structurally cycloparaffinic
1146. The most widely employed screening technique for drug abuse is?
a. High performance liquid chromatography
b. Gas liquid chromatography
c. Thin layer chromatography ✔
d. UV spectrophotometry
1147. Phenobarbital is a metabolite of?
a. Primidone ✔
b. Phenytoin
c. Amobarbital
d. Secobarbital
1148. Increased serum albumin concentrations are seen in which of the following conditions?
a. Nephrotic syndrome
b. Acute hepatitis
c. Chronic inflammation
d. Dehydration ✔
1149. Which of the following amino acids is associated with sulfhydryl group?
a. Cysteine ✔
b. Glycine
c. Serine
d. Tyrosine
1150. Night blindness is associated with deficiency of which of the following vitamins?
a. A ✔
b. C
c. Niacin
d. Thiamine
1151. Scurvy is associated with deficiency of which of the following vitamins?
a. A
b. C ✔
c. Niacin
d. Thiamine
1152. A cardiac glycoside that is used in the treatment of congenital heart failure and arrhythmias by increasing the
force and velocity of myocardial contraction is?
a. Digoxin ✔
b. Acetaminophen
c. Lithium
d. Phenytoin
1153. Pellagra is associated with deficiency of which of the following vitamins?
a. A
b. C
c. Niacin ✔
d. Thiamine
1154. A drug that relaxes the smooth muscles of the bronchial passages is?
a. Acetaminophen
b. Lithium
c. Phenytoin
d. Theophylline ✔
1155. Which of the following is the formula for calculating the unknown concentration based on Beer’s Law (A=
absorbance, C= Concentration)?
a. (A unknown/A standard) x C standard ✔
b. C standard x A unknown
c. A standard x A unknown
d. (C standard)/(A standard) x 100
1156. Which of the following is the formula for coefficient of variation?
a. (standard deviation x 100)/standard error
b. (mean x 100)/standard deviation
c. (standard deviation x 100)/mean ✔
d. (variance x 100)/mean
1157. Rickets is associated with deficiency in which of the following vitamins?
a. A
b. C
c. Niacin
d. D ✔
1158. The regulation of calcium and phosphorus metabolism is accomplished by which of the following glands?
a. Thyroid
b. Parathyroid ✔
c. Adrenal glands
d. Pituitary
1309. All of the following are associated with hemolytic anemia except:
a. Methemoglobinemia
b. Hemoglobinuria
c. Hemoglobinemia
d. Increased haptoglobin ✔
1310. Autoimmune hemolytic anemia is best characterized by which of the following?
a. Increased levels of plasma C3
b. Spherocytic red cells ✔
c. Decreased osmotic fragility
d. Decreased unconjugated bilirubin
1311. “Bite cells” are usually seen in patients with:
a. Rh null trait
b. Chronic granulomatous disease
c. G6PD deficiency ✔
d. PK deficiency
1312. The morphological classification on anemias is based on which of the following:
a. M:E ratio
b. Prussian blue stain
c. RBC indicies ✔
d. Reticulocyte count
1313. Which of the following is a common finding in aplastic anemia?
a. A monoclonal disorder
b. Tumor infiltration
c. Peripheral blood pancytopenia ✔
d. Defective DNA synthesis
1314. Congenital dyserythropoietic anemias (CDAs) are characterized by:
a. Bizarre multinucleated erythroblasts ✔
b. Cytogenetic disorders
c. Megaloblastic erythropoiesis
d. An elevated M:E ratio
1315. Microangiopathic hemolytic anemia is characterized by:
a. Target cells and cabot rings
b. Toxic granulation and Dohle bodies
c. Pappenheimer bodies and basophilic stippling
d. Schistocytes and nucleated RBCs ✔
1316. Which antibiotics are most often implicated in the development of aplastic anemia?
a. Sulfonamides
b. Penicillin
c. Tertrcycline
d. Chloramphenicol ✔
1317. Sickle cell disorders are:
a. Hereditary, intracorpuscular RBC defect ✔
b. Hereditary, extracorpuscular RBC defect
c. Acquired, intracorpuscular RBC defects
d. Acquired, extracorpuscular RBC defects
1318. Which of the following conditions may produce spherocytes in a peripheral smear?
a. Pelger-Huet anomaly
b. Pernicious anemia
c. Autoimmune hemolytic anemia ✔
d. Sideroblastic anemia
1319. A patient’s peripheral smear reveals numerous NRBC’s marked variation of red cell morphology, and pronounced
polychromasia. In addition to a decreased Hgb and decreased Hct values, what other CBC parameters may be
anticipated?
a. Reduced platelets
b. Increased MCHC
c. Increased MCV ✔
d. Decreased red cell distribution width (RDW)
1320. What red cell inclusions may be seen in the peripheral blood smear of a patient postsplenectomy?
a. Toxic granulation
b. Howell-Jolly bodies ✔
c. Malarial parasite
d. Siderotic granules
1321. Reticulocytosis usually indicates:
a. Response to inflammation
b. Neoplastic process
c. Aplastic anemia
d. Red cell regeneration ✔
1322. Hereditary pyropoikilocytosis (HP) is a red cell membrance defect characterized by:
a. Increased pencil-shaped cells
b. Increased oval macrocytes
c. Misshappen budding fragmented cells ✔
d. Bite cells
1323. Following overnight fasting, hypoglycemia in adults is defined as glucose of:
a. A <_70 mg/dl (<_3.9 mmol/L)
b. B <_60 mg/dl (<3.3 mmol/L)
c. C <_55 mg/dl (<_3.0 mmol/L)
d. D <_45 mg/dl (<2.5 mmol/L) ✔
1324. The preparation of a patient for standard for glucose tolerance testing should include:
a. A high carbohydrate diet for 3 days ✔
b. A low carbohydrate diet for 3 days
c. Fasting for 48 hours prior to testing
d. Bed rest for 3 days
1325. If a fasting glucose was 90 mg/dl, which of the following 2hr postprandial glucose results would most closely
represent normal glucose metabalism
a. 55 mg/dl (3.0 mmol/L)
b. 100 mg/dl (5.5 mmol/L) ✔
c. 180 mg/dl (9.9 mmol/L)
d. 260 mg/dl (14.3 mmol/L)
1326. A healthy person with a blood glucose of 80 mg/dl (4.4 mmol/L) would have a simultaneously determined
cerebrospinal fluid glucose value of:
a. 25 mg/dl (1.4 mmol/L)
b. 50 mg/dl (2.3 mmol/L) ✔
c. 100 mg/dl (5.5 mmol/L)
d. 150 mg/dl (8.3 mmol/L)
1327. Cerebrospinal fluid for glucose assay should be:
a. Refrigerated
b. Analyzed immediately ✔
c. Heated to 56 degrees celsius
d. Stored at room temperature
1328. Which of the following 2 hr postprandial glucose values demonstrates unequivocal hyperglycemia diagnostic for
biabetes mellitus
a. 160 mg/dl (8.8 mmol/L)
b. 170 mg/dl (9.4 mmol/L)
c. 180 mg/dl (9.9 mmol/L)
d. 200 mg/dl (11. mmol/L) ✔
1329. Serum levels that define hypoglycemia in pre-term or low birth weight infants are:
a. The same as adults
b. Lower than adults ✔
c. The same as a normal full-term infant
d. Higher than a normal full-term infant
1330. The conversion of glucose or other hexoses into lactate or pyruvate is called:
a. Glycogenesis
b. Glycolysis ✔
c. Gluconeogenesis
d. Glycogenolysis
1331. Which of the following values obtained during a glucose tolerance test are diagnosticof diabetes mellitus?
a. 2hr specimen= 150 mg/dl
b. Fasting plasma glucose= 126 mg/dl ✔
c. Fasating plasma glucose= 110 mg/dl
d. 2hr specimen = 180 mg/dl
1332. The glycated hemoglobin value represents the integrated values of glucose concentration during the preceding:
a. 1-3 weeks
b. 4-5 weeks
c. 6-8 weeks ✔
d. 16-20 weeks
1333. Monitoring long-term glucose control in patients with adult onset diabetes mellitus can best be accomplished by
measuring:
a. Weekly fasting 7am serum glucose
b. Glucose tolerance testing
c. 2hr postprandial serum glucose
d. Hemoglobin A1c ✔
1334. The glycosylated hemoglobin levels in a hemolysate reflect the:
a. Average blood glucose levels of the past 2-3 months ✔
b. Average blood glucose levels for the past week
c. Blood glucose level at the time the sample drawn
d. Hemoglobin a1c level at the time the sample is drawn
1335. Which of the following hemoglobins has a glucose-6-phosphate on the amino-terminal valine of the beta chain:
a. S
b. C
c. A2
d. A1c ✔
1336. A person with hemolytic anemia will:
a. Show a decrease in glycated hgb value ✔
b. Show an increase in glycated hgb value
c. Show lil or no change in glycated hgb value
d. Demonstrate an elevated hgb A1
1337. In using ion-exchange chromotographic methods, falsely increased levels of hgb A1c might be demonstrated in
the presence of:
a. Iron defiency anemia
b. Pernicious anemia
c. Thalassemias
d. Hgb S ✔
1338. An increase in serum acetone is indicative of a defect in the metabolism of:
a. Carbohydrates ✔
b. Fat
c. Urea nitrogen
d. Uric acid
1339. What is the best method to diagnose lactase deficiency?
a. H2 breath test ✔
b. Plasma adolase lvl
c. LDH level
d. D-xylose test
1340. The expected blood gas results for a patient in chronic renal failure would match the pattern of:
a. Metabolic acidosis ✔
b. Respiratory acidosis
c. Metabolic alkalosis
d. Respiratory alkalosis
1341. Severe diarrhea causes
a. Metabolic acidosis ✔
b. Repiratory acidosis
c. Metabolic alkalosis
d. Respiratory alkalosis
1342. Factors that contribute to a PCO2 electrode requiring 60-120 seconds to reach equilibrium include the:
a. Diffusion characteristics of the membrane ✔
b. Actual blood pO2
c. Type of calibrating standard (ie, liquid or humidified gas)
d. Potential of the polarizing mercury cell
1343. An emphysema patient suffering from fluid accumulation in the alveolar spaces is the likely to be in what
metabolic state?
a. Respiratory acidosis ✔
b. Respiratoy alkalosis
c. Metabolic acidosis
d. Metabolic alkalosis
1344. At blood ph 7.40, what is the ratio of bicarbonate to carbonic acid?
a. 15:1
b. 20:1 ✔
c. 25:1
d. 30:1
1345. The referance range for the pH of arterial blood measured at 37 degree celsius is:
a. 7.28-7.34
b. 7.33-7.37
c. 7.35-7.45 ✔
d. 7.45-7.50
1346. Metabolic acidosis is described as a (n):
a. Increase in CO2 content and PCO2 with a decreased pH ✔
b. Decrease in CO2 content with an increased pH
c. Increase in CO2 with an increased pH
d. Decrease in CO2 content and PCO2 with a decreased pH
1347. A common cause of respiratory alkalosis is:
a. Vomiting
b. Starvation
c. Asthma
d. Hyperventilation ✔
1348. Acidosis and alkalosis are best defined as fluctuations in blood pH and CO2 content due to changes in:
a. Bohr effect
b. O2 content ✔
c. Bicarbonate buffer
d. Carbonic anhydrase
1349. Select the test which evaluates renal tubular function:
a. IVP
b. Creatinine clearance
c. Osmolarity ✔
d. Microscopic urinalysis
1350. The degree to which the kidney concentrates the glomerular filtrate can be determined by:
a. Urine creatine
b. Serum creatine
c. Cratinine clearance
d. Urine to serum osmolality ratio ✔
1351. Osmolal gap is aa difference between:
a. The ideal and real osmolality values
b. Calculated and measured osmolality values ✔
c. Plasma and water osmolality values
d. Molality and molarity at 4 degres celsius
1352. The most important buffer pair in plasma is the
a. Phosphate/biphosphate pair
b. Hemoglobin/imidazole pair
c. Bicarbonate/carbonic acid pair ✔
d. Sulfate/bisulfate pair
1353. Quantitation of Na+ and K+ by ion-selective electrode is the standard method because:
a. Dilution is required for flame photometry
b. There is no lipoprotein interference
c. Of advances in electrochemistry ✔
d. Of the abscence of an internal standard
1354. Most of the carbon dioxide present in blood in the form of:
a. Dissolved CO^2
b. Carbonate
c. Bicarbonate ion ✔
d. Carbonic acid
1355. Serum “anion gap” is increased in patients with:
a. Renal tubular acidosis
b. Diabetic alkalosis
c. Metabolic acidosis due to diarrhea
d. Lactic acidosis ✔
1356. The anion gap is useful for quality control of laboratory results for:
a. Amino acids and proteins
b. Blood gas analyses
c. Sodium, potassium, chloride, and total CO2 ✔
d. Calcium, phosphorus, and magnesium
1357. The buffering capacity of blood is maintained by a reversible exchange process between bicarbonate and:
a. Sodium
b. Potassium
c. Calcium
d. Chloride ✔
1358. In respiratory acidosis, a compensatory mechanism is the increase in :
a. Respiratory rate
b. Ammonia formation
c. Blood PCO2
d. Plasma bicarbonate concentration ✔
1359. Which of the following electrolytes is the chief plasma cation whose main fuction is maintaining osmotic
pressure?
a. Chloride
b. Calcium
c. Potassium
d. Sodium ✔
1360. The solute that contributes the most to the total serum osmolality is:
a. Glucose
b. Sodium ✔
c. Chloride
d. Urea
1361. Which of the following is true about direct ion selective electrode is falsely decreased by:
a. Whole blood specimens are acceptable ✔
b. Elevated lipids cause falsely decreased results
c. Elevated proteins cause falsely decreased results
d. Elevated platelets cause falsely increased results
1362. Sodium determination by indirect ion selective electrode is falsely decreased by:
a. Elevated chloride levels
b. Elevated lipid levels ✔
c. Decreased protein levels
d. Decreased albumin levels
1363. Which percentage of total serum calcium is nondiffusible protein bound?
a. 80%-90%
b. 51%-60%
c. 40%-50% ✔
d. 10%-30%
1364. Calcium concentration in the serum is regulated by:
a. Insulin
b. Parathyroid hormone ✔
c. Thyroxine
d. Vitamin C
1365. The regulation of calcium and phosphorus metabolism is accomplished by which of the following glands?
a. Thyroid
b. Parathyroid ✔
c. Adrenal glands
d. Pituitary
1366. A hospitalized patient is experiencing increased neuromuscular irritability (tetany). Which of the following tests
should be ordered immediately?
a. Calcium ✔
b. Phosphate
c. BUN
d. Glucose
1367. Which is the following is most likely to be ordered in addition to serum calcium to determine the cause of tetany?
a. Magnesium ✔
b. Phosphate
c. Sodium
d. Vitamin
1368. A reciprocal relationship exists between:
a. Sodium and potassium
b. Calcium and phosphate ✔
c. Chloride and CO2
d. Calcium and magnesium
1369. Fasting serum phosphate concentration is controlled primarily by the:
a. Pancreas
b. Skeleton
c. Parathyroid glands ✔
d. Small intestine
1370. A low concentration of serum phosphorus is commonly found in:
a. Patients who are receiving carbohydrate hyperalimentation ✔
b. Chronic renal disease
c. Hypoparathyroidism
d. Patients with pituitary tumors
1371. The primary function serum albumin in the peripheral blood is to:
a. Maintain colloidal osmotic pressure ✔
b. Increase antibody production
c. Increase fibrinogen formation
d. Maintain blood viscosity
1372. The first step in analyzing a 24-hour urine specimen for quantitative urine protein is:
a. Subculture the urine for bacteria
b. Add the appropriate preservative
c. Screen for albumin using a dipstick
d. Measure the total volume ✔
1373. Total iron-binding capacity measures the serum iron transporting capacity of :
a. Hemoglobin
b. Cerruloplasmin
c. Transferrin ✔
d. Fetrritin
1374. The first step in the quantitation of serum iron is:
a. Direct reaction with appropriate chromagen
b. Iron saturation of transferrin
c. Free iron precipitation
d. Separation of iron from transferring ✔
1375. A serum sample demonstrates an elevated result when tested with the Jaffe reaction. This indicates:
a. Prolonged hypothermia
b. Renal functional impairment ✔
c. Pregnancy
d. Arrhythmia
1376. Creatinine clearance is used to estimate the:
a. Tubular secretion of creatinine
b. Glomerular secretion of creatinine
c. Renal glomerular and tubular mass
d. Glomerular filtration rate ✔
1377. 90% of the copper present in the blood is bound to:
a. Transferrin
b. Ceruloplasmin ✔
c. Albumin
d. Cryoglobulin
1378. Hemoglobin S can be separated from hemoglobin D by:
a. Electrophoresis on a diffirent medium and acidic pH ✔
b. Hemoglobin A2 quantitation
c. Electrophoresis at higher voltage
d. Kleihauer-Betke acid elution
1379. On electrophoresis at alkaline pH, which of the following is the slowest migrating hemoglobin?
a. Hgb A
b. Hgb S
c. Hgb C ✔
d. Hgb F
1380. Urobilinogen is formed in the:
a. Kidney
b. Spleen
c. Liver
d. Intestine ✔
1381. Kernicterus is an abnormal accumulation of bilirubin in:
a. Heart tissue
b. Brain tissue ✔
c. Liver tissue
d. Kidney tissue
1382. A stool specimen that appears black and tarry should be tested for the presence of:
a. Occult blood ✔
b. Fecal fat
c. Trypsin
d. Excess mucus
1383. The most specific enzyme test for acute pancreatitis is:
a. Acid phosphatase
b. Trypsin
c. Amylase
d. Lipase ✔
1384. Which of the following enzymes are used in the diagnosis of acute pancreatitis?
a. Amylase (AMS) and lipase (LPS) ✔
b. Aspartate aminotransferase (AST) and alanine aminotransferase (ALT)
c. 5′ -nucleotidase (5’N) and gamma-glutamyl transferase (GGT)
d. Aspartate aminotransferase (AST) and lactate dehydrogenase(LD)
1385. Which of the following enzymes catalyzes the conversion of starch to glucose and maltose?
a. Malate dehydrogenase (MD)
b. Amylase (AMS) ✔
c. Creatine kinase (CK)
d. Isocitric dehydrogenase (ICD)
1386. Aspartate amino transferase (AST) is characteristically elevated in diseases of the:
a. Liver ✔
b. Kidney
c. Intestine
d. Pancreas
1387. Aspartate aminotransferase (AST) and alanine aminotransferase (ALT ) are both elevated in which of the
following diseases ?
a. Muscular dystrophy
b. Viral hepatitis ✔
c. Pulmonary emboli
d. Infectious mononucleosis
1388. The greatest activities of serum AST and ALT are seen in which of the following?
a. Acute viral hepatitis
b. Primary biliary cirrhosis ✔
c. Metastatic hepatic cirrhosis
d. Alcoholic cirrhosis
1389. Which of the following clinical disorders is associated with the greatest elevation of lactate dehydrogenase
isoenzyme 1?
a. Pneumonia
b. Glomerulonephritis
c. Pancreatitis
d. Pernicious anemia ✔
1390. The enzyme present in almost all tissues that may be separated by electrophoresis into 5 components is:
a. Lipase
b. Transaminase
c. Creatine kinase
d. Lactate dehydrogenase ✔
1391. A common cause of falsely increased LD1 fraction of lactic dehydrogenase is:
a. Specimen hemolysis ✔
b. Liver disease
c. Congestive heart failure
d. Drug toxicity
1392. The presence of which of the following isoenzymes indicates acute myocardial damage?
a. CKMM
b. CKMB ✔
c. CKBB
d. None
1393. In which of the following conditions would a normal level of creatine kinase be found?
a. Acute myocardial infarct
b. Hepatitis ✔
c. Progressive muscular dystrophy
d. Intramuscular injection
1394. Of the following diseases, the one most often associated with elevations of lactate dehydrogenase isoenzymes 4
and 5 on electrophoresis is:
a. Liver disease ✔
b. Hemolytic anemia
c. Myocardial infarction
d. Pulmonary edema
1395. When myocardial infarction occurs, the first enzyme to become elevated is:
a. CK ✔
b. LD
c. AST
d. ALT
1396. In the immunoinhibition phase of the CKMB procedure:
a. M subunit is inactivated ✔
b. B subunit is inactivated
c. MB is inactivated
d. BB is inactivated
1397. The presence of increased CKMB activity on a CK electrophoresis pattern is most likely found in a patient
suffering from:
a. Acute muscular stress following strenuous exercise
b. Malignant liver disease
c. Myocardial infarction ✔
d. Severe head injury
1398. Increaseed serum lactic dehydrogenase activity due to elevation of fast fraction (1 and 2) on elctrophoretic
seperation is caused by:
a. Nephrotci syndrome
b. Hemolytica anemia ✔
c. Pancreatitis
d. Hepatic damage
1399. Regan isoenzyme has the same properties as alkaline phosphatase that orginates in the
a. Skeleton
b. Kidney
c. Intestine
d. Placenta✔
1400. The most heat labile fraction of alkaline phosphatase is obtained from:
a. Liver
b. Bone ✔
c. Intestine
d. Placenta
1401. The most sensitive enzymatic indicator for liver damage from ethanol intake is:
a. Alanine aminotransferase (ALT)
b. Aspartate aminotransferase (AST)
c. Gamma-glutamyl transferase (GGT) ✔
d. Alkaline phosphatase
1402. Isoenzyme assays are performed to improve:
a. Precision
b. Accuracy
c. Sensitivity
d. Specificity ✔
1403. The protein portion of an enzyme complec is called the:
a. Apoenzyme ✔
b. Coenzyme
c. Holoenzyme
d. Proenzyme
1404. Which of the following chemical determinations may be of help in establishing the presence of seminal fluid?
a. Lactic dehydrogenase (LD)
b. Isocitrate dehydrogenase (ICD)
c. Acid phosphatase ✔
d. Alkaline phosphatase
1405. Chylomicrons are present in which of the following dylipidemias?
a. Familial hypercholoesterolemia
b. Hypertriglyceridemia
c. Deficiency in lipoprotein lipase activity ✔
d. Familial hypoalphalipoprotein
1406. Turbidity in serum siggest elevation of:
a. Cholesterol
b. Total protein
c. Chylomicrons ✔
d. Albumin
1407. TSH is prouced by the:
a. Hypothalamus
b. Pituitary gland ✔
c. Adrenal cortex
d. Thyroid
1408. The majority if thyroxine (T4) is converted into the more biologically active hormone:
a. Thyroglubulin
b. Thyroid-stimulating hormone (TSH)
c. Triiodothyronine ✔
d. Thyrotropoin-releasing hormone
1409. The screening test for congenital hypothyroidism is based upon:
a. TSH level in the newborn
b. Thyroid-binding glubulin level in the newborn
c. Iodine level in the newborn
d. Al thyroxine (t4) level in the newborn ✔
1410. Which of the following is secreted by the placents and used for the early detection of pregnancy?
a. Luteinizing hormone (LH)
b. Human chronic gonadotropin (HCG) ✔
c. Follicle-stimulating hormone (FSH)
d. Progesterone
1411. In amniotic fluid, the procedure used to detect hemolytic disease of the newborn is:
a. Measurement of absorbance at 450 nm ✔
b. Creatinine
c. Lecithin/sphingomyelin ratio
d. Estriol
1412. During a normal pregnancy, quantitative human chorionic gonadotropin levels peak at how many weeks after the
last menstrual period?
a. 2-4
b. 8-10 ✔
c. 14-16
d. 18-20
1413. Whi the following steroids is an adrenal cortical hormone?
a. Angiotensinogen
b. Aldosterone ✔
c. Epinephrine
d. Growth hormone
1414. What common substrate is used in the biosynthesis of adrenal steroids, including androgens and estrogens?
a. Cortisol
b. Catecholamines
c. Progesterone
d. Cholesterol ✔
1415. The major action of angiotensin II is:
a. Increased pituitary secretion of vasopressin
b. Increased parathormone secretion by the parathyroid
c. Decreased adrenal secretion of aldosterone
d. Increased vasoconstriction ✔
1416. The urinary excretion product measured as an indicator of epinephrine production is:
a. Dopamine
b. Dihydroxyphenylalanine (DOPA)
c. Homovanillic acid
d. Vanillymandelic acid (VMA) ✔
1417. Which of the following hormones regulates normal blood calcium levels?
a. Thyroxine
b. Estriol
c. Parathyroid hormone ✔
d. Growth hormone
1418. A diagnosisof primary adrenal insufficiency requires demonstration of:
a. Decreased urinary 17-keto and 17-hydroxysteroids
b. Decreased cortisol production
c. Impaired response to ACTH stimulation ✔
d. Increased urinary cortisol excretion after metyrapone
1419. The screen for adrenal cortical hyperfunction with the greatest sensitivity and specificity is:
a. A 24 hour free cortisol ✔
b. Plasma cortisol
c. Urinary 17-hydroxycorticosteroids
d. Plasma corticosterone
1420. Aldosterone is released by the adrenal cortex upon stimulation by:
a. Renin
b. Angiotensinogen
c. Angiotensin I
d. Angiotensin II ✔
1421. Clinical assays for tumor markers are most important for:
a. Screeening for the presence of cancer
b. Monitoring the course of a known cancer ✔
c. Confirming the abscence of disease
d. Identifying patients at risk for cancer
1422. Detection of which of the following substances is most useful to monitor the course of a patient with testicular
cancer?
a. Alpha-fetoprotein ✔
b. Carcinoembryonic antigen
c. Prolactin
d. Testosterone
1423. Increased concentrations of alpha-fetoprotein (AFP) in adults are most characteristically associated with:
a. Hepatocellular carcinoma ✔
b. Alcoholic cirrhosis
c. Chronic active hepatitis
d. Multiple myeloma
1424. Carcinoembryonic antigen (CEA) is most likely to be produced in a malignany involving the:
a. Brain
b. Testes
c. Colon ✔
d. Bone
1425. Which of the following is useful in the detection and management of carcinoma of the prostate?
a. Total prostate-specific antigen ✔
b. Prostatic acid phosphatase
c. Human chronic gonadotropin
d. Alpha-fetoprotein
1426. Which of the following statements most correctly describes the utility of clinical laboratory assays for tumor
markers?
a. Tumor markers are useful to screen asymptomatic patients for tumors
b. Tumor markers are highly specific
c. Tumor markers indicate the likelihgood of an indvidual developing a tumor
d. Tumor markers are useful in tracking the effcacy of treatment ✔
1427. Cancer antigen 125 (CA 125) is a tumor marker associated with:
a. Breast carcinoma
b. Colon cancer
c. Lung cancer
d. Ovarian and endometrial carcinoma ✔
1428. In addition to carcinoma of the prostate, elevated prostate-specific antigen (PSA) can occur due to:
a. Aspirin therapy
b. Exogenous steroid use
c. Benign prostatic hyperplasia ✔
d. Statin therapy (cholesterol lowering drug)
1429. Blood glucose levels are directly regulated by the hormone
a. Hydrocortisone
b. Insulin ✔
c. Thyroxin
d. ACTH
1430. According to Landsteiner, when a specific antigen is present on blood cells, the corresponding antibody
a. May or may not be present depending on the agglutinogens present
b. Is present in the serum
c. Is present in the blood cells
d. Is absent from the serum ✔
1431. Metabolic acidosis can be detected by testing urine for the presence of
a. Glucose
b. Uric acid
c. Ketone bodies ✔
d. Protein
1432. The major cation found in the extra cellular fluid is
a. Chloride
b. Bicarbonate
c. Sodium ✔
d. Potassium
1433. Who is credited with processing the most readily acceptable theory of ABO inheritance?
a. Levine
b. Landsteiner ✔
c. Weiner
d. Bernstein
1434. Safranin in a Gram stain is used as a
a. Primary stain
b. Secondary stain ✔
c. Mordant
d. Decolorizer
1435. Antihuman serum globulin reagent
a. Is produced in laboratory animals ✔
b. Is produced in humans
c. Never detects complement-dependent antibodies
d. Occurs naturally in most humans
1436. Strepococcus pneumoniae
a. Grows best at slightly acid pH
b. Is motile
c. Is consistently gram-positive even in old cultures
d. Capsules are produced by virulent strians ✔
1437. The type of anemia usually associated with severe burns is
a. Macrocytic
b. Microcytic
c. Aplastic
d. Hemolytic ✔
1438. A substance that produces a prolonged prothrombin time when given orally is
a. Heparin
b. Protamine sulfate
c. Saliclate
d. Coumadin ✔
1439. The principle involved in some automated blood cell counters is based on the
a. Amount of hemoglobin in the red cell
b. Size of the particle being counted ✔
c. Weight of the hemoglobin in the red cell
d. Value of the cell indices
1440. On an automated blood cell counter, the two parameters affected by a high background count would be
a. Hct and WBC
b. RBC and WBC ✔
c. RBC and Hgb
d. WBC and Hgb
1441. Leukemia may be suspected when a manual hematocrit determination reveals
a. Hemolysis
b. A heavy buffy coat ✔
c. Icteric plasma
d. A high hematocrit
1442. A mother is Rh(D) negative. The father is homozygous Rh(D) positive. All of their offspring will be
a. Homozygous Rh(D) positive
b. Heterozygous Rh(D) positive ✔
c. 50% Rh(D) positive and 50% Rh(D)negative
d. Erythroblastotic
1443. The quantity of inactivated serum used for qualitative VDRL test is
a. 0.05 mL ✔
b. 0.10 mL
c. 0.15 mL
d. 0.02 mL
1444. Hansel’s stain is appropriate for
a. Phagocytic neutrophils
b. Leukocytes in spinal fluid
c. Nasal secrection for eosinophiles ✔
d. Circulating eosinophiles
1445. As a general rule, when a blood cell matures
a. The cell decreases in size ✔
b. There is no change in the cell’s size
c. The nucleus increases in size
d. The cell increases in size
1446. An RBC exhibiting hypochromia would be described as being
a. Packed with hemoglobin
b. Variable in shape
c. Markedly bluish in color ✔
d. Markedly pale in central color
1447. Freezing point depression measurements are part of which one of the following urine test procedures?
a. Refractive index
b. Osmolality ✔
c. Hydrometry
d. Specific gravity
1448. When performing automated cell counts, most automated cell counted instruments
a. Count nucleated red blood cells with leukocytes ✔
b. Count nucleated red blood cells with platelets
c. Do not count nucleated red blood cells
d. Count nucleated red blood cells with erythrocytes
1449. The egg of the Schistosoma characterized by a pronounced lateral spine is
a. Schistosoma mansoni ✔
b. Schistosoma haematobium
c. Schistosoma japonicum
d. Schistosoma hepatica
1450. Fibrinogen determinations are performed on
a. Serum only
b. Plasma only ✔
c. Either serum or plasma
d. Any body fluid
1451. A fladellate frequently found in the urine of female patients is
a. Trichomonas tenax
b. Trichomonas hominis
c. Trichomonas vaginalis ✔
d. Entamoeba coli
1452. Unconjugated bilirubin is also known as
a. Biliverdin
b. Total bilirubin
c. Conjugated bilirubin
d. Prehepatic bilirubin ✔
1453. According to Landsteiner, when a specific antigen is present on blood cells, the corresponding antibody
a. May or may not be present depending on the agglutinogens present
b. Is present in the serum
c. Is present in the blood cells
d. Is absent from the serum ✔
1454. A reactive fluorescent treponemal antibody (FTA-AB) test
a. Indicates no infection
b. Is positive during the chancre stage
c. Confirms the presence of treponemal antibodies ✔
d. Indicates the severity of infection
1455. The screenign or presumptive test for the osmotic fragility of red cells is normal when hemolysis begins in
a. 0.50% NaCl ✔
b. 0.90% NaCl
c. 1.34% NaCl
d. 0.85% NaCl
1456. Most methods for the determination of blood creatinine are based on the reaction of creatinine and
a. Sulfuric acid
b. Alkaline picrate
c. Ammonium hydroxide
d. Acetic anhydride
1457. Hansel’s stain is appropriate for
a. Nasal secrection for eosinophiles ✔
b. Circulating eosinophiles
c. Phagocytic neutrophils
d. Leukocytes in spinal fluid
1458. Brilliant cresyl blue or new methylene blue are stains used for counting
a. Platelets
b. Reticulocytes ✔
c. Howell-Jolly bodies
d. Malaria
1459. Supravital staining of red cells with a deficiency of G-6-PD will demonstrate the presence of
a. Rubriblasts
b. Howell-Jolly bodies
c. Heniz bodies ✔
d. Plasmodium species
1460. A donor who recently tested positive for HBsAg should be deferred
a. For 5 years
b. For 1 year
c. For 6 months
d. Permanently ✔
1461. The end-product of purine metabolism is
a. Uric acid ✔
b. Creatine
c. Creatinine
d. Urea
1462. Water-soluble pigments are produced by
a. Yeasts
b. Staphylococcus epidermidis
c. Pseudomonas aeruginosa ✔
d. Group A streptococcus
1463. Alkaline phosphatase is GREATLY elevated in
a. Kidney disease
b. Obstructive jaundice ✔
c. Liver disease
d. Myocardial infarction
1464. A floatation method for concentration of ova and cysts used
a. Zinc chloride
b. Concentrated formalin
c. Ammonium sulfate
d. Zinc sulfate ✔
1465. The organism that can cause rheumatic fever and/or glomerular nephritis is
a. Staphylococcus aureus
b. Staphylococcus haemolyticus
c. Streptococcus pyogenes ✔
d. Streptococcus viridans
1466. In the formation of urine, the function of the glomerulus is
a. Secretion
b. Re-absorption of water
c. Selective re-absorption
d. Simple filtration ✔
1467. Metabolic acidosis can be detected by testing urine for the presence of
a. Protein
b. Uric acid
c. Glucose
d. Ketone bodies ✔
1468. A substance that produces a prolonged prothrombin time when given orally is
a. Protamine sulfate
b. Coumadin ✔
c. Heparin
d. Saliclate
1469. Which one of the following is characteristic of any antigen?
a. Produced by action of antibody
b. High molecular weight
c. Foreign to animal
d. High order of specificity ✔
1470. A variety of media may be safely stored for months is care is taken to
a. Retain their moisture ✔
b. Avoid exposing them to light
c. Maintain them at room temperature
d. Maintain them in an incubator
1471. The ketone bodies include acetoacetic acid, acetone, and
a. Acetic acid
b. Oxaloacetic acid
c. Lactic acid
d. 3-hydroxy butyric acid ✔
1472. The accepted and usual time and temperature used for the inactivation of serum is
a. 25 C for 1 hour
b. 37 C for 30 min
c. 56 C for 30 min ✔
d. 56 C for 10 min
1473. When using white blood cell pipets for performing a white cell count, blood is diluted
a. 1:200
b. 1:10
c. 1:50
d. 1:20 ✔
1474. The degree that a procedure deviates from a known value or from a calculated mean value is known as
a. Stardard deviation ✔
b. Coefficient variation
c. Percent deviation
d. Quality control
1475. Which of the following tests is specific for urinary glucose?
a. Dip stick ✔
b. Benedict’s
c. Clinitest
d. Pandy
1476. Dilute normal urine is usually
a. Dark yellow
b. Reddish-yellow
c. Amber
d. Pale yellow ✔
1477. The principle involved in some automated blood cell counters is based on the
a. Amount of hemoglobin in the red cell
b. Value of the cell indices
c. Weight of the hemoglobin in the red cell
d. Size of the particle being counted ✔
1478. All of the following influence glomerular filtration EXCEPT
a. Hyperglycemia and renal tubule malfunction ✔
b. Decreased renal blood flow and cardiac failure
c. Renal or urinary calculi and decreased renal blood flow
d. Cardiac failure and renal or urinary calculi
1479. To determine if a patient is A1 or A2, the blood is typed with
a. Dolichos biflorus serum ✔
b. Anti-A serum
c. Anti-AB serum
d. Anti-A2 serum
1480. Who is credited with processing the most readily acceptable theory of ABO inheritance?
a. Levine
b. Bernstein
c. Weiner
d. Landsteiner ✔
1481. On most automated cell counted, background counts are made using
a. Lysing reagent only
b. Diluting fluid ✔
c. Distilled water
d. Highly-diluted blood
1482. Blood group A individuals have
a. Anti-B in their serum ✔
b. Antigen A and B on their red cells
c. Anti-A in their serum
d. Anti-O in their serum
1483. The ketone test area on a dip stick is impregnated with
a. Alkaline copper
b. Ferric chloride
c. 2,4 dichloraniline
d. Nitroprusside ✔
1484. Which of the following tests is used to measure capillary fragility?
a. Prothrombin time
b. Bleeding time
c. Partial thromboplastin time (PTT)
d. Tourniquet ✔
1485. Most of the plasma thyroxine (T4) is
a. Free
b. Bound to cholesterol
c. Bound to globulin ✔
d. Bound to albumin
1486. The etiologic agent of chancroid is
a. Haemophilus aegyptius
b. Bordetella pertussis
c. Haemophilus influenzae
d. Haemophilus ducreyi ✔
1487. WHich one of the following is a function of gamma globulin?
a. Transports glucose
b. Regulates body temperature
c. Performs as fibrinogen for blood coagulation
d. Provides humoral immunity ✔
1488. During the maturation of a blood cell, the nuclear chromatin pattern becomes
a. Finer
b. More acidic
c. Less dense
d. More dense ✔
1489. Cerebral spinal fluid patients with post-cerebral hemmorrhage appears
a. Light yellow or straw colored ✔
b. Bright red
c. Greeen
d. Clear (colorless)
1490. Freezing point depression measurements are part of which one of the following urine test procedures?
a. Hydrometry
b. Refractive index
c. Osmolality ✔
d. Specific gravity
1491. The infective stage of the hookworm is the
a. Rhabditiform larva with a short buccal cavity
b. Filariform larva with a notched tail
c. Filariform larva with a pointed tail ✔
d. Rhabditiform larva with a long buccal cavity
1492. Enterobius vermilcularis is a
a. Hookworm
b. Flat worm
c. Filarial worm
d. Pinworm ✔
1493. Sodium is responsible for the maintenance of
a. Blood coagulation
b. Osmotic pressure of body fluids ✔
c. Cardiac muscle contractions
d. Salt intake
1494. In Taenia saginata, the larval stage develops in
a. Man
b. Swine
c. Fish
d. Cattle ✔
1495. When performing automated cell counts, most automated cell counted instruments
a. Count nucleated red blood cells with leukocytes ✔
b. Do not count nucleated red blood cells
c. Count nucleated red blood cells with erythrocytes
d. Count nucleated red blood cells with platelets
1496. Antihuman serum globulin (Coombs) is NOT used in performing
a. Reverse typing ✔
b. Immunoglobulin testing
c. D(u) testing
d. Autoagglutination tests
1497. The mother of a 4-year-old child notes that her child is sleeping poorly and scratching his anal area. You suspect
the child may have pinworms. Which one of the following is the BEST method to make that diagnosis?
a. Examine the stool for the presence of trophozoites
b. Determine the titer of IgE antibody against the organism
c. Examine a blood smear for the presence of microfilaria
d. Examine transparent adhesive tape for the presence of eggs ✔
e. Examine the stool for the presence of cysts
1498. Larval stage of Taenia saginata is called
a. hydatid cyst
b. cysticercus cellulosae
c. cysticercoid
d. cysticercus bovis ✔
1499. A 4-year-old boy presents to his pediatrician with intense perianal itching. His mother explains that the child has
also been extremely irritable during the day and has not been sleeping well at night. Eggs with a flattened side
were identified by the laboratory technician from a piece of scotch tape brought in by the parent. Infection with
which of the following organisms is most likely?
a. Ascaris lumbricoides
b. Entamoeba histolytica
c. Echinococcus granulosus
d. Trichuris trichiura
e. Enterobius vermicularis ✔
1500. Four weeks after his arrival from Africa, a 24-year-old graduate student presents with blood in his urine.
Microscopic examination of his urine reveals the presence of eggs with terminal spines. In the interview he admits
that he has been working on his family’s rice field occasionally since his early childhood. The most likely
etiologic agent of his complaint is
a. Schistosoma japonicum
b. Entamoeba histolytica
c. Schistosoma haematobium ✔
d. Fasciolopsis buski
e. Schistosoma mansoni
1501. Each of the following parasites is transmitted by eating inadequately cooked fish or seafood EXCEPT:
a. Clonorchis sinensis
b. Ancylostoma duodenale ✔
c. Paragonimus westermani
d. Diphyllobothrium latum
1502. Each of the following parasites is transmitted by eating inadequately cooked fish or seafood EXCEPT:
a. Clonorchis sinensis
b. Ancylostoma duodenale
c. Paragonimus westermani ✔
d. Diphyllobothrium latum
1503. Each of the following statements concerning neisseriae is correct EXCEPT:
a. They produce IgA protease as a virulence factor.
b. They are oxidase-positive.
c. They are gram-negative diplococci.
d. They grow best under anaerobic conditions. ✔
1504. Several pathogens are transmitted either during gestation or at birth. Which one of the following is LEAST likely
to be transmitted at these times?
a. Haemophilus influenzae ✔
b. Treponema pallidum
c. Neisseria gonorrhoeae
d. Chlamydia trachomatis
1505. A 30-year-old woman with systemic lupus erythematosus is found to have a positive serologic test for syphilis
(VDRL test). She denies having had sexual contact with a partner who had symptoms of a venereal disease. The
next best step would be to:
a. Perform a fluorescent treponemal antibody-absorbed (FTA-ABS) test on a specimen of her serum ✔
b. Reassure her that the test is a false-positive reaction related to her autoimmune disorder
c. Trace her sexual contacts for serologic testing
d. Treat her with penicillin
1506. Each of the following is associated with the Lancefield group B streptococci (S. agalactiae) EXCEPT:
a. Vaginal carriage in 5% to 25% of normal women of childbearing age
b. Neonatal sepsis and meningitis
c. Pyoderma (impetigo) ✔
d. Beta-hemolysis
1507. Each of the following statements concerning gonorrhea is correct EXCEPT:
a. Gonococcal conjunctivitis of the newborn rarely occurs in the United States, because silver nitrate or
erythromycin is commonly used as prophylaxis.
b. A presumptive diagnosis can be made by finding gram-negative kidney bean-shaped diplococci within
neutrophils in a urethral discharge.
c. Infection in men is more frequently symptomatic than in women.
d. The definitive diagnosis can be made by detecting antibodies to Neisseria gonorrhoeae in the patient’s
serum. ✔
1508. Each of the following statements concerning chlamydial genital tract infections is correct EXCEPT:
a. There is no vaccine against these infections.
b. Infection can persist after administration of penicillin.
c. Symptomatic infections can be associated with urethral or cervical discharge containing many
polymorphonuclear leukocytes.
d. Infection can be diagnosed by finding antichlamydial antibody in a serum specimen. ✔
1509. Each of the following statements concerning Actinomyces and Nocardia is correct EXCEPT:
a. A. israelii is an anaerobic rod found as part of the normal flora in the mouth.
b. Infections are usually diagnosed by detecting a significant rise in antibody titer. ✔
c. Both Actinomyces and Nocardia are branching, filamentous rods.
d. N. asteroides causes infections primarily in immunocompromised patients.
1510. Each of the following statements concerning the fluorescent treponemal antibody-absorbed (FTA-ABS) test for
syphilis is correct EXCEPT:
a. The patient’s serum is absorbed with saprophytic treponemes.
b. The test is rarely positive in primary syphilis. ✔
c. Once positive, the test remains so despite appropriate therapy.
d. The test is specific for Treponema pallidum.
1511. Regarding Chlamydiae, which one of the following is MOST accurate?
a. They replicate in the nucleus of infected cells, where they form inclusions that are useful diagnostically.
b. Their life cycle consists of a metabolically inactive particle in the extracellular phase. ✔
c. They exhibit swarming motility on a blood agar plate.
d. They are gram-positive rods that do not form spores.
e. They can replicate only within cells because they lack the ability to produce certain essential mRNAs.
1512. Regarding human papillomavirus (HPV), which one of the following is MOST accurate?
a. HPV induces the formation of koilocytes in the skin that are an important diagnostic feature of HPV
infection. ✔
b. Amantadine is a chain-terminating drug that inhibits HPV replication by blocking DNA synthesis.
c. HPV is an enveloped virus with a genome composed of double-stranded RNA.
d. The P2 capsid protein of HPV activates the c-sarc oncogene in human cells, which is the process by
which HPV predisposes to malignancy
e. Blood and blood products are an important mode of transmission of HPV.
1513. A 30-year-old woman presents to her gynecologist with complaints of vaginal itching and a frothy, yellow
discharge. She also complains of painful urination. She admits to being sexually active with several men in the
past two weeks. Cultures are negative for bacterial growth, but organisms are visible via a wet preparation on low
power. The most likely causal agent is
a. Trichomonas vaginalis ✔
b. Candida albicans
c. Chlamydia trachomatis
d. Trichophyton rubrum
e. Giardia lamblia
1514. Each of the following statements concerning Chlamydia trachomatis is correct EXCEPT:
a. It is an important cause of nongonococcal urethritis.
b. It is an important cause of subacute bacterial endocarditis. ✔
c. It is the cause of lymphogranuloma venereum.
d. It is an important cause of conjunctivitis.
1515. A 20-year-old woman presents with a history of vaginal discharge for the past 3 days. On pelvic examination, you
see a mucopurulent exudate at the cervical os, and there is tenderness on palpation of the right fallopian tube. You
do a Gram stain and culture on the cervical discharge. The culture is done on Thayer-Martin medium. Of the
following, which findings are the MOST likely to be found?
a. A Gram stain reveals many neutrophils and gram-variable rods, and culture on Thayer-Martin medium
reveals ß-hemolytic colonies.
b. A Gram stain reveals many neutrophils but no gram-negative diplococci are seen, and culture on Thayer-
Martin medium reveals coagulase-positive colonies.
c. A Gram stain reveals many neutrophils and spirochetes, and culture on Thayer-Martin medium reveals no
colonies.
d. A Gram stain reveals many neutrophils and gram-negative diplococci, and culture on Thayer-Martin
medium reveals oxidase-positive colonies. ✔
1516. Regarding human immunodeficiency virus (HIV), which one of the following is MOST accurate?
a. Both zidovudine and lamivudine block HIV replication by inhibiting cleavage of the precursor
polypeptide by the virion-encoded protease.
b. The Western blot test for antibodies to HIV has more false-positive results than the ELISA test.
c. The antigenicity of the GAG protein of HIV is highly variable, which is a significant impediment to the
development of a vaccine against HIV.
d. The term viral load refers to the concentration of HIV RNA in the patient’s blood plasma. ✔
1517. Each of the following statements concerning chlamydiae is correct EXCEPT:
a. Chlamydia trachomatis has multiple serotypes that can cause different diseases.
b. Chlamydiae are strict intracellular parasites because they cannot synthesize sufficient adenosine
triphosphate (ATP).
c. Most chlamydiae are transmitted by arthropods. ✔
d. Chlamydiae possess both DNA and RNA and are bounded by a cell wall.
1518. Which of the following statements about Transmission Electron Microscopy is not true.
a. The beam is focused by electromagnetic lenses.
b. The specimen must be stained with osmium or other heavy metal. ✔
c. The specimens are placed in a high vacuum for viewing.
d. The specimens must be sliced very thin, 20-100 nm in thickness.
1519. Phase Contrast microscopy
a. Continuously changes the phase of the incident light from the condenser to improve contrast in the
specimen.
b. Uses circular filters in the condenser and objective to give contrast to parts of the cell with different
refractive indices. ✔
c. Uses special lenses to change the color of light passing through them.
d. Uses special lenses to distinguish between solid and liquid phases of the cell.
1520. Place the structures of the compound light microscope in the order that light passes through them on the way to
the observer’s eyes: (1) condenser, (2) ocular lens, (3) illuminator, (4) specimen, (5) objective lens.
a. 2-1-4-5-3
b. 3-5-4-1-2
c. 2-5-4-1-3
d. 3-1-4-5-2 ✔
e. 3-4-1-5-2
1521. Most light microscope contain a/an…. that converges the light beam so that it passes through the specimen
a. objective lens
b. mechanical stage
c. ocular lens
d. condenser ✔
e. iris diaphragm
1522. Light of… wavelength typically will result in…. resolving power.
a. shorter, worse
b. any, poor
c. longer, better
d. shorter, better ✔
1523. “Parfocal” refers to microscopes with multiple objectives where
a. objectives are used in pairs for stereoscopic effects.
b. sequential objectives increase power by a factor of two.
c. each objective is positioned to be in focus at the same stage height. ✔
d. each objective has the same working distance above the specimen.
1524. Regarding the Gram stain, which one of the following is the MOST accurate?
a. If you forget to stain with the red dye (safranin or basic fuchsin), both gram-positive bacteria and gram-
negative bacteria will appear blue.
b. One reason why bacteria have a different color in this stain is because the gram-positive bacteria have
lipid in their membrane, whereas gram-negative bacteria do not.
c. After adding crystal violet and Gram’s iodine, both gram-positive bacteria and gram-negative bacteria
will appear blue. ✔
d. If you forget to heat-fix, both gram-positive bacteria and gram-negative bacteria will appear blue.
1525. Negative staining is useful for observing the presence of ..
a. Endospores
b. Motility
c. Mycolic acid
d. Flagella
e. Capsule ✔
1526. In this type of microscopy, surface of a specimen is bombarded with electrons during sample preparation step.
a. Phase-contrast Microscope
b. Scanning Electron Microscope ✔
c. Transmission Electron Microscope
d. Bright-light Microscope
e. Dark field Microscope
1527. When a microscopic image is brightly illuminated, but is surrounded by a black field, what type of microscope is
being used?
a. Dark-field ✔
b. Bright-field
c. Fluorescence
d. Phase-contrast
e. Electron
1528. Monochromatic light is sometimes used to increase the resolution of light microscopes. Light of which color
below would give you the best resolution?
a. Green
b. Blue ✔
c. Orange
d. Red
1529. While viewing the stained slides in 100x, a drop of oil is placed between the tip of the oil immersion lens and the
specimen on a glass slide. What is the purpose of using the oil.
a. The oil increases the degree of refraction.
b. The oil prevents light from bending as it passes through the specimen. ✔
c. The oil increases the total power of magnification.
d. The oil decreases the numerical aperture.
e. The oil helps to illuminate the specimen.
1530. Differential Interference Contrast microscopy
a. compares two identical specimens on the same microscope.
b. illuminates the specimen with light of two different colors.
c. illuminates the specimen with both reflected and transmitted light.
d. illuminates the specimen with light of two different phases. ✔
1531. Each of the following statements concerning malaria is correct EXCEPT:
a. Early in infection, sporozoites enter hepatocytes.
b. Release of merozoites from red blood cells causes periodic fever and chills.
c. The female Anopheles mosquito is the vector.
d. The principal site of gametocyte formation is the human gastrointestinal tract. ✔
1532. Black water fever is a special manifestation of malaria caused by;
a. P. falciparum ✔
b. P. malariae
c. P. vivax
d. P. ovale
1533. All of the following statements about mosquitoes are true except:
a. Only female mosquitoes sucks human blood.
b. They are definitive host in Filaria ✔
c. Its life cycle is completed in 3 weeks
d. They are definitive host in Malaria
1534. Malaria is transmitted by a bite of female Anopheles mosquito, but what causes the disease?
a. A Bacteria
b. A parasite ✔
c. An yeast
d. A virus
1535. Which one of the following agents can be used to prevent malaria?
a. Chloroquine ✔
b. Praziquantel
c. Mebendazole
d. Inactivated vaccine
1536. Which of the following mosquito is responsible for malaria transmission?
a. Anopheles ✔
b. Culex tritaeniorhynchus
c. Aedes albopictus
d. Aedes aegypti
1537. Which of the following statement(s) regarding Plasmodium falciparum are true?
a. causes benign tertian malaria
b. is associated with recurrent relapses after initial treatment because of liver hypnozoites
c. is the only malarial parasite causing greater than 20% parasitaemia ✔
d. is the only cause of cerebral malaria
1538. The word “malaria” comes from which two medieval Italian words?
a. Evil insect
b. Painful breathing
c. Bad air ✔
d. Dirty water
1539. Can malaria be transmitted from person to person?
a. Yes, you can get malaria touching a person with malaria
b. Yes, you can get malaria sharing space with malaria patients
c. No, except from mother to child during pregnancy ✔
d. Yes, you can get malaria through kissing
1540. After sporozoite gain entrance to human body it undergoes developmental cycle first in liver than in RBC, only
after which fever is seen. This incubation period varies between plasmodium species, and ………….. species has
longest incubation period.
a. P. ovale
b. P. vivax
c. P. falciparum ✔
d. P. malariae
1541. In malaria, the form of plasmodia that is transmitted from mosquito to human is the:
a. Hypnozoite
b. Merozoite
c. Sporozoite ✔
d. Gametocyte
1542. A 35-year-old man comes to the emergency department complaining of high fever, chills, severe headache, and
confusion. He has recently returned from Africa. A peripheral blood smear reveals multiple ring structures and
crescent-shaped gametes. Which of the following organisms is the most likely cause?
a. Plasmodium falciparum ✔
b. Plasmodium vivax
c. Plasmodium malariae
d. Plasmodium ovale
e. Leishmania species
1543. Children at day care centers in the United States have a high rate of infection with which one of the following?
a. Enterobius vermicularis ✔
b. Necator americanus
c. Entamoeba histolytica
d. Ascaris lumbricoides
1544. Each of the following statements concerning Strongyloides stercoralis is correct EXCEPT:
a. S. stercoralis produces filariform larvae.
b. Migrating larvae of S. stercoralis induce a marked eosinophilia.
c. S. stercoralis is acquired by ingestion of eggs. ✔
d. S. stercoralis undergoes a free-living life cycle in soil.
1545. Each of the following statements concerning hydatid cyst disease is correct EXCEPT:
a. The disease is caused by Echinococcus granulosus.
b. The disease is caused by a parasite whose adult form lives in dogs’ intestines.
c. The disease occurs primarily in tropical Africa. ✔
d. The cysts occur primarily in the liver.
1546. The skin test used for the diagnosis of Hydatid cyst is known as:
a. Schick test
b. Casoni test ✔
c. Montenegro test
d. Dick test
1547. A 13-year-old boy from India was brought to the emergency room with a prolapsed rectum. Examination of the
rectum reveals small worms that resemble whips attached to the mucosa. A stool sample reveals eggs that are
barrel shaped, with bipolar plugs. Which of the following is the most likely cause?
a. Ascaris lumbricoides
b. Entamoeba histolytica
c. Echinococcus granulosus
d. Trichuris trichiura ✔
e. Enterobius vermicularis
f. Giardia lamblia
1548. Each of the following parasites passes through the lung during human infection EXCEPT:
a. Strongyloides stercoralis
b. Wuchereria bancrofti ✔
c. Ascaris lumbricoides
d. Necator americanus
1549. Larval stage of Taenia solium is called
a. cysticercus cellulosae ✔
b. hydatid cyst
c. cysticercus bovis
d. cysticercoid
1550. At a school nurse’s request, a clinic in rural South Carolina sees a 9-year-old girl who appears listless and
inattentive, although hearing and visual testing has been within normal limits. The physician finds the child thin,
with the “potbelly” of malnutrition, and orders a fecal exam and CBC. The CBC reveals a microcytic,
hypochrornic anemia, and the fecal exam detects brown, oval nematode eggs approximately 65 microns in size,
too numerous to count. What was the most likely means by which this child was infected?
a. Ingestion of ova
b. Ingestion of cysts in muscle
c. Skin penetration by larvae ✔
d. Ingestion of larvae
e. Mosquito transmission of sporozoites
1551. Pigs or dogs are the source of human infection by each of the following parasites EXCEPT:
a. Ascaris lumbricoides
b. Taenia solium
c. Trichinella spiralis ✔
d. Echinococcus granulosus
1552. Each of the following parasites is transmitted by flies EXCEPT:
a. Loa loa
b. Trypanosoma gambiense
c. Onchocerca volvulus
d. Schistosoma mansoni ✔
1553. Each of the following statements concerning Schistosoma haematobium is correct EXCEPT:
a. S. haematobium infection predisposes to bladder carcinoma.
b. S. haematobium eggs have no spine. ✔
c. Snails are intermediate hosts of S. haematobium.
d. S. haematobium is acquired by humans when cercariae penetrate the skin.
1554. Each of the following statements concerning hookworm infection is correct EXCEPT:
a. Hookworm infection is acquired by humans when filariform larvae penetrate the skin.
b. Hookworm infection can cause anemia.
c. Hookworm infection can be diagnosed by finding the trophozoite in the stool. ✔
d. Hookworm infection is caused by Necator americanus.
1555. Each of the following statements concerning Ascaris lumbricoides is correct EXCEPT:
a. Both dogs and cats are intermediate hosts of A. lumbricoides. ✔
b. A. lumbricoides is one of the largest nematodes.
c. A. lumbricoides is transmitted by ingestion of eggs.
d. A. lumbricoides can cause pneumonia.
1556. Which of the following tapeworms is acquired by eating raw or under-cooked pork?
a. Hymenolepis nana
b. Taenia solium ✔
c. Taenia saginata
d. Diphyllobothrium latum
1557. The main anatomic location of Schistosoma mansoni adult worms is:
a. Lung alveoli ✔
b. Intestinal venules
c. Renal tubules
d. Bone marrow
1558. How many magnifying lenses does a compound light microscope contain?
a. Two ✔
b. One
c. Three
d. Four
1559. In light microscopes, what function does a condenser serve?
a. Increases light intensity
b. Focuses the light rays on the sample ✔
c. Reduces glare
d. Focuses the light rays onto our eyes
e. Magnifies the light rays after their passage through the sample
1560. The order of reagents used in the Gram stain are:
a. Crystal violet, iodine, alcohol, safranin ✔
b. Alcohol, crystal violet, iodine, safranin
c. Crystal violet, safranin, alcohol, iodine
d. Crystal violet, iodine, safranin, alcohol
e. Iodine, crystal violet, safranin, alcohol
1561. The compound light microscope can be used to observe:
a. Bacteria, cell organelles, and red blood cells ✔
b. Amino acids, bacteria, and red blood cells
c. Ribosomes, bacteria, cell organelles, and red blood cells
d. Atoms, proteins, viruses, and bacteria
e. Viruses, bacteria, cell organelles, and red blood cells
1562. All of the following are examples of special stains except:
a. Flagellar stain
b. Ziehl-Neelsen acid-fast stain
c. Negative stains ✔
d. Endospore stain
1563. Which of the following statements is most correct about the differential Gram stain?
a. Acetone/Alcohol differentially destains Gram negative cells. ✔
b. Crystal violet differentially stains Gram positive cells.
c. Safranin differentially stains Gram negative cells.
d. Gram’s iodine differentially stains Gram positive cells.
1564. The total magnification of a microscope is calculated by:
a. The objective lens power squared
b. Multiplication of the objective lens and ocular lens magnification powers ✔
c. Multiplication of the objective lens and condenser lens magnification powers
d. Addition of the objective lens and ocular lens magnification powers
1565. Which of the following stains is used to classify microorganisms based on their cell wall content?
a. Gram stain ✔
b. Methylene blue
c. Capsular stain
d. Spore stain
e. Negative stain
1566. Which of the following microscopic techniques provide three-dimensional images of a bacterial cell?
a. Dark-field microscopy
b. Fluorescent microscopy
c. Transmission electron microscopy
d. Scanning electron microscopy ✔
1567. Each of the following statements concerning Mycobacterium tuberculosis is correct EXCEPT:
a. M. tuberculosis appears as a red rod in Gram-stained specimens. ✔
b. After being stained with carbolfuchsin, M. tuberculosis resists decolorization with acid alcohol.
c. M. tuberculosis has a large amount of mycolic acid in its cell wall.
d. M. tuberculosis appears as a red rod in acid-fast stained specimens.
1568. Which of the following is defined as the ability to distinguish or separate two adjacent objects or points
from one another in a microscopic specimen?
a. Reflection
b. Illuminate
c. Image
d. Resolving power ✔
e. Refraction
1569. The presence of a capsule around bacterial cells usually indicates their increased disease-causing potential
and resistance to disinfection. Capsules are generally viewed by:
a. Scanning electron microscopy
b. Negative staining ✔
c. Ziehl-Neelsen staining
d. Gram staining
1570. Drumstick appearance in the microscopy is the distinguishing characteristics of:
a. Clostridium tetani ✔
b. Clostridium perfringens
c. Clostridium difficile
d. Clostridium botulinum
1571. What type of microscopy allows for the visualization of internal components within live, unstained
specimens?
a. Phase-contrast ✔
b. Fluorescence
c. Electron
d. Bright-field
e. Dark-field
1572. Transmission electron microscopy is best for high magnification viewing of
a. surface structure of fixed cells
b. surface membranes of live, motile cells.
c. internal structure of live, motile cells.
d. internal structure of fixed cells. ✔
1573. Your patient is a 30-year-old woman with nonbloody diarrhea for the past 14 hours. Which one of the
following organisms is LEAST likely to cause this illness?
a. Clostridium difficile
b. Salmonella enteritidis
c. Streptococcus pyogenes ✔
d. Shigella dysenteriae
1574. Each of the following agents is a recognized cause of diarrhea EXCEPT:
a. Enterococcus faecalis ✔
b. Escherichia coli
c. Clostridium perfringens
d. Vibrio cholerae
1575. Which one of the following organisms that infects the gastrointestinal tract is the MOST frequent cause of
bacteremia?
a. Salmonella typhi ✔
b. Pyrazinamide
c. Streptomycin
d. Rifampicin
1576. Which one of the following bacteria has the LOWEST 50% infectious dose (ID50)?
a. Campylobacter jejuni
b. Shigella sonnei ✔
c. Salmonella typhi
d. Vibrio cholerae
1577. A family has a diarrhea after eating raw shellfish due to an oxidase positive, gram-negative curved bacillus
that requires a special media for growth. Which of the following is the most likely cause?
a. Staphylococcus aureus
b. Clostridium botulinum
c. Escherichia coli
d. Vibrio parahaemolyticus ✔
1578. A 26-year-old male with AIDS presents with painful bloody diarrhea. Stool culture on MacConkey agar
medium and subsequent analyses reveals predominantly a gram -negative facultative anaerobic rod, which
is oxidase negative, lactose negative, H2S negative and non-motile. Which of the following is the most likely
cause of this infection?
a. Shigella flexneri ✔
b. Vibrio cholerae
c. Enterobacter aerogenes
d. Enterotoxigenic Escherichia coli
e. Salmonella enteritidis
1579. Which of the following pathogenic bacteria is MOST COMMONLY associated with traveler’s diarrhea?
a. Shigella sonnei
b. Salmonella typhimurium
c. Yersinia enterocolitica
d. Escherichia coli ETEC (enterotoxingenic). ✔
e. Campylobacter jejuni
1580. Diarrhea associated with prolonged antibiotic treatment, including pseudomembranous colitis, is MOST
FREQUENTLY caused by
a. Yersinia enterocolitica
b. Salmonella enteritidis
c. Clostridium difficile ✔
d. Shigella dysenteriae
e. Rota Virus
1581. A 25-year-old female medical student reports that she had lunch at a Indian restaurant in the afternoon.
She is presented in the emergency department of hospital with the complaints of abdominal cramps, nausea
and vomiting. Culture and gram stain reveal gram- positive aerobic rods. Which of the following is the
most likely cause of this infection?
a. Staphylococcal food poisoning
b. Bacillus cereus food poisoning ✔
c. Clostridium perfringens food poisoning
d. Clostridium botulinum food poisoning
1582. A specimen suspected of containing members of the Enterobacteriaceae was subjected to a few laboratory
diagnostic tests and generated the following results: Gram negative rod; colorless/pale colonies on
MacConkey agar. All of the following microorganisms may be included in the diagnosis, EXCEPT
a. Shigella spp
b. Salmonella spp
c. Both Salmonella and Shigella
d. Escherichia coli ✔
1583. Five hours after eating reheated rice at a restaurant, a 24-year-old woman and her husband both developed
nausea, vomiting, and diarrhea. Which one of the following organisms is the MOST likely to be involved?
a. Bacillus cereus
b. Salmonella typhi
c. Clostridium perfringens
d. Enterotoxigenic Escherichia coli
1584. A 29-year-old man with a long history of heartburn and dyspepsia presents to the primary care doctor with
severe mid-epigastric pain. He was initially treated with an H2 blocker twice a day. He immediately had
relief but his symptoms returned after two weeks. What is the most likely cause for the relapse in his
symptoms?
a. Outlet obstruction due to fibrous tissue repair
b. Giardia lamblia infection
c. Helicobacter pylori infection ✔
d. Acute gastric ulcerations
e. Tapeworm infestation
1585. For which one of the following enteric illnesses is a chronic carrier state MOST likely to develop?
a. Campylobacter enterocolitis
b. Shigella enterocolitis
c. Cholera
d. Typhoid fever ✔
1586. The pathogenesis of which one of the following organisms is MOST likely to involve invasion of the
intestinal mucosa?
a. Enterotoxigenic Escherichia coli
b. Clostridium botulinum
c. Shigella sonnei ✔
d. Vibrio cholerae
1587. A 70-year-old man underwent bowel surgery for colon cancer 3 days ago. He now has a fever and
abdominal pain. Attending physician is concerned that he may have peritonitis. Which one of the following
pairs of organisms is MOST likely to be the cause?
a. Bordetella pertussis and Salmonella enteritidis
b. Clostridium botulinum and Shigella dysenteriae
c. Actinomyces israelii and Campylobacter jejuni
d. Bacteroides fragilis and Klebsiella pneumoniae ✔
1588. Which one of the following host defense mechanisms is the MOST important for preventing dysentery
caused by Salmonella?
a. Salivary enzymes
b. Normal flora of the mouth
c. Alpha interferon
d. Gastric acid ✔
1589. The following organisms are recognized causes of diarrhea EXCEPT
a. Salmonella
b. Rotavirus
c. Enterovirus
d. Campylobacter
e. Enterobacter ✔
1590. A local day care center has several simultaneous cases of diarrhea. A methylene blue fecal smear was taken
during the outbreak from one of the children and revealed numerous PMN’s and occult blood. Culture of
the stool revealed a typical gram negative rod which fermented glucose but not lactose, was nonmotile and
did not produce H2S. The agent which MAY have caused this outbreak was
a. Escherichia coli
b. Salmonella typhimurium
c. Campylobacter jejuni
d. Rota Virus
e. Shigella sonnei ✔
1591. A 27-year-old IV drug user presents with difficulty swallowing. Examination of the oropharynx reveals
white plaques along the tongue and the oral mucosa. Which of the following best describes the microscopic
appearance of the microorganism responsible for this patient’s illness?
a. Mold with septate hyphae
b. Encapsulated yeast
c. Budding yeast and pseudohyphae ✔
d. Mold with nonseptate hyphae
1592. Two hours after a delicious Thanksgiving dinner of barley soup, roast turkey, stuffing, sweet potato, green
beans, cranberry sauce, and pumpkin pie topped with whipped cream, the Smith family of four experience
vomiting and diarrhea. Which one of the following organisms is MOST likely to cause these symptoms?
a. Campylobacter jejuni
b. Salmonella enteritidis
c. Shigella flexneri
d. Staphylococcus aureus ✔
1593. Following specialized transduction, if any of the bacterial genes transferred in are to be stabilized, what
process must occur?
a. Generalized transduction
b. Conjugation
c. Site-specific recombination
d. Transformation
e. Specialized transduction
f. Homologous recombination ✔
1594. Which genetic material is found in pathogenic Corynebacterium diphtheriae but not in nonpathogenic
normal flora diphtheroids?
a. Highly repetitive bacterial DNA
b. A diphthamide on eEF-2
c. An F factor
d. An integrated temperate phage ✔
e. An episome
1595. Which one of the following processes involves a sex pilus?
a. Conjugation resulting in transfer of an R (resistance) factor ✔
b. Transduction of a chromosomal gene
c. Integration of a temperate bacteriophage
d. Transposition of a mobile genetic element
1596. How is a prophage created?
a. Through site-specific recombination of a temperate phage and bacterial DNA ✔
b. Through infection of a bacterial cell with lambda phage, lacking the lambda repressor
c. Through activation of the recA gene product of an exogenote
d. Through excision of bacterial DNA and active lytic replication of a bacteriophage
e. Through infection of a bacterial cell with a virulent bacteriophage
1597. Which of the following events is MOST likely to be due to bacterial conjugation?
a. A gene encoding resistance to gentamicin in the Escherichia coli chromosome appears in the genome of a
bacteriophage that has infected E. coli.
b. An encapsulated strain of Streptococcus pneumoniae acquires the gene for capsule formation from an
extract of DNA from another encapsulated strain.
c. A strain of Pseudomonas aeruginosa produces ß-lactamase encoded by a plasmid similar to a plasmid of
another gram-negative organism. ✔
d. A strain of Corynebacterium diphtheriae produces a toxin encoded by a prophage.
1598. The process by which bacterial or plasmid DNA may be mistakenly incorporated (during assembly) into
one phage being produced by the lytic life cycle and then that DNA-transferred to another bacterial cell
which may acquire some new genetic traits is called
a. Specialized transduction
b. Conjugation
c. Homologous recombination
d. Generalized transduction ✔
e. Transformation
1599. Which of the following mechanisms is most likely to be involved in multiple drug resistance transfer from
one cell to another?
a. Transformation of chromosomal genes
b. Transposition
c. Specialized transduction of a chromosomal gene for drug resistance
d. Conjugation with a cell with a free plasmid carrying drug resistance ✔
1600. Conjugation with a cell with chromosomal drug resistance appears in the genome of a bacteriophage that
has infected it.
a. is the conversion of a virulent phage into a temperate phage.
b. is a change in pathogenicity due to the presence of a prophage. ✔
c. is the immunity that a prophage confers on a bacterium.
d. refers to the incorporation of a prophage into the chromosome.
e. is the induction of a prophage to its virulent state.
1601. Which of the following events is most likely due to bacterial transformation?
a. A formerly non-toxigenic strain of Corynebacterium diphtheriae becomes toxigenic.
b. A gene for gentamicin resistance from an Escherichia coli chromosome
c. A strain of Neisseria gonorrhoeae starts producing a plasmid-encoded beta-lactamase similar to that
another Gram-negative strain.
d. A non-encapsulated strain of Streptococcus pneumoniae acquires a gene for capsule formation from the
extract of an encapsulated strain. ✔
1602. Which of the following statements concerning recombinant DNA technology is false?
a. Recombinant DNA technology provides a high degree of risk to the health of the general population. ✔
b. Thus far, no illnesses in laboratory workers have been traced to genetic recombinants.
c. Recombinant DNA technology offers specific benefits to the scientific, medical, and general population.
d. Mutant strains of bacteria produced by genetic recombination are often unable to survive in the natural
environment.
e. Production of large amounts of proteins such as insulin and human growth hormone has been made
possible using recombinant DNA technology.
1603. The ability of a cell to bind DNA to its surface and import it is required for which genetic process?
a. Transformation ✔
b. Homologous recombination
c. Conjugation
d. Specialized transduction
e. Generalized transduction
1604. Hybridomas are formed by fusing antibody-producing white blood cells with:
a. Bone marrow cells
b. Viruses
c. Red blood cells
d. Bacteria
e. Myeloma cells ✔
1605. Which type of genetic exchange in bacteria is susceptible to the activity of deoxyribonuclease?
a. Transduction
b. Transformation ✔
c. Transfection
d. Conjugation
1606. The form of genetic exchange in which donor DNA is introduced to the recipient by a bacterial virus is
a. Transformation
b. Horizontal transfer
c. Transfection
d. Conjugation
e. Transduction ✔
1607. The formation of a mating pair during the process of conjugation in Escherichia coli requires
a. Restriction endonuclease
b. Transfer of both strands of DNA
c. sex pilus ✔
d. Intergration of Transposon
e. Lysis of the donor
1608. The first vaccine for human use produced using recombinant DNA technology was the:
a. AIDS vaccine
b. Polio vaccine
c. Hepatitis A vaccine
d. Hepatitis B vaccine ✔
e. MMR vaccine
1609. Each of the following events involves recombinantion of DNA EXCEPT:
a. Transduction of a chromosomal gene
b. Integration of a temperate bacteriophage
c. Transposition of a mobile genetic element
d. Conjugation, eg, transfer of an R (resistance) factor ✔
1610. Mutations in katG gene of Mycobacterium tuberculosis are responsible for resistance to: isoniazid
(INH)11XDR TB is defined as
a. Strain resistant to more than one drug (excluding co resistance to INH and rif)
b. resistant to two most effective first-line therapeutic drugs, isoniazid and rifampin irrespective of
resistance to any other drug
c. MDR TB that also is resistant to fluoroquinolones and at least one of the three injectable second-line
drugs used to treat TB (amikacin, kanamycin, or capreomycin) ✔
d. MDR TB that is resistant to either one of the injectable second line drugs or second-line therapeutic
fluoroquinolones
1611. A case of leprosy with good Cell mediated immunity usually presents as following form of disease
a. Tuberculoid ✔
b. Lepromatous
c. Indeterminate
d. Borderline
1612. Which of the following environmental factors do NOT increase the probability that M tuberculosis will be
transmitted?
a. Inadequate local or general ventilation that results in insufficient dilution or removal of infectious droplet
nuclei.
b. Improper specimen handling procedures that generate infectious droplet nuclei.
c. Recirculation of air containing infectious droplet nuclei.
d. Negative pressure in an infectious TB patient’s room. ✔
e. Exposure in small enclosed spaces.
1613. Tubercular Granulomas are made up of
a. damaged tissue cells and bacilli only
b. fibrous and damaged tissue and bacilli only
c. organized aggregates of immune cells that surround foci of infected tissues ✔
d. necrotic tissue and damaged tissue and bacilli only
1614. MDR TB is defined as resistance to
a. more than three anti tubercular drugs
b. fluoroquinolones and at least one of the three injectable second-line drugs used to treat TB
c. Isoniazid, Pyrazinamide and Rifampicin
d. isoniazid and rifampin irrespective of resistance to any other drug ✔
1615. Swimming pool granuloma is caused by
a. M. chelonei
b. M. marinum ✔
c. M. tuberculosis
d. M. ulcerancs
1616. Cord growth seen on mycobacterial culture is characteristic of
a. Saprophytic strains
b. Virulent strains ✔
c. Avirulent strains
d. Atypical mycobacteria
1617. Buruli ulcer is caused by
a. M. marinum
b. M. chelonei
c. M. ulcerancs ✔
d. M. tuberculosis
1618. The scientist who discovered M. tuberculosis was:
a. Louis Pasteur
b. Jean-Antoine Villemin
c. Robert Koch ✔
d. Calmette and Guerin
1619. The mother of a 4-year-old child notes that her child is sleeping poorly and scratching his anal area. You
suspect the child may have pinworms. Which one of the following is the BEST method to make that
diagnosis?
a. Examine a blood smear for the presence of microfilaria
b. Examine the stool for the presence of cysts
c. Determine the titer of IgE antibody against the organism
d. Examine the stool for the presence of trophozoites
e. Examine transparent adhesive tape for the presence of eggs ✔
1620. Each of the following statements concerning Schistosoma haematobium is correct EXCEPT:
a. S. haematobium infection predisposes to bladder carcinoma.
b. S. haematobium is acquired by humans when cercariae penetrate the skin.
c. Snails are intermediate hosts of S. haematobium.
d. S. haematobium eggs have no spine. ✔
1621. A 34-year-old woman presents to her physician with complaints of midepigastric pain. She describes the
pain as moderate, occasionally waking her at night, and improving immediately following meals. A urease
breath test was positive. Which of the following correctly describes the causal agent?
a. Gram-negative rod; aerobic
b. Gram-negative rod; facultative anaerobe
c. Gram-negative curved rod; microaerophilic 11
d. Gram-positive rod; aerobic
e. Gram-positive rod; microaerophilic
1622. Cholera toxin effects are mediated by stimulation of which of the following second messengers:
a. cGMP
b. Ca++ calmodulin
c. cAMP 11
d. IP3/DAG
1623.
1624. An infant presents to the emergency department due to difficulty breathing, constipation, and anorexia.
Upon examination, the physician notes flaccid paralysis. A toxin screen of the stool identified the agent.
What is the mechanism of action of the toxin?
a. ADP ribosylation of eukaryotic elongation factor 2 (eEF-2)
b. ADP ribosylation of Gi
c. Blocks release of inhibitory transmitters GABA and glycine
d. ADP ribosylation of GTP-binding protein
e. Blocks release of acetylcholine ✔
1625. A group of 6 college students undertake to climb Mt. Rainier outside Seattle on their spring break. They
pack food and camping provisions except for water, which they obtain from the many fresh water
mountain streams that arise at the summit. The adventure takes a little over a week to accomplish, and all
return safely in good spirits to their classes the following week. Within the first week after their return, 5 of
the 6 students report to the infirmary with profuse diarrhea and tenesmus. Each affected student
experiences weakness and weight loss, and stool samples submitted to the lab are yellow, greasy, and foul
smelling. What attribute of this parasite imparts its pathogenicity?
a. Encystment
b. Lytic enzymes
c. Toxic metabolites
d. Flagella
e. Ventral sucking disc ✔
1626. Each of the following statements concerning Strongyloides stercoralis is correct EXCEPT:
a. S. stercoralis produces filariform larvae.
b. Migrating larvae of S. stercoralis induce a marked eosinophilia.
c. S. stercoralis is acquired by ingestion of eggs. ✔
d. S. stercoralis undergoes a free-living life cycle in soil.
1627. Each of the following statements concerning exotoxins is correct EXCEPT:
a. Botulism is caused by a toxin that hydrolyzes lecithin (lecithinase), thereby destroying nerve cells. ✔
b. Some strains of Escherichia coli produce an enterotoxin that causes diarrhea.
c. Cholera toxin acts by stimulating adenylate cyclase.
d. Diphtheria is caused by an exotoxin that inhibits protein synthesis by inactivating an elongation factor.
1628. Each of the following statements concerning Ascaris lumbricoides is correct EXCEPT:
a. A. lumbricoides can cause pneumonia.
b. Both dogs and cats are intermediate hosts of A. lumbricoides. ✔
c. A. lumbricoides is transmitted by ingestion of eggs.
d. A. lumbricoides is one of the largest nematodes.
1629. Which of the following ‘Vibrios’ is most commonly associated with ear infections:
a. V. parahemolyticus
b. V. fulnifcus
c. Vibrio fluvialis
d. V. alginolyticus ✔
1630. A 4-year-old boy presents to his pediatrician with intense perianal itching. His mother explains that the
child has also been extremely irritable during the day and has not been sleeping well at night. Eggs with a
flattened side were identified by the laboratory technician from a piece of scotch tape brought in by the
parent. Infection with which of the following organisms is most likely?
a. Echinococcus granulosus
b. Trichuris trichiura
c. Enterobius vermicularis ✔
d. Ascaris lumbricoides
e. Entamoeba histolytica
1631. An HIV-positive patient with a CD4+ count of 47 presents with diarrhea. Acidfast structures are found in
the stool. From this finding, which of the following is true?
a. Even with the best treatment, the infection may be unrelenting ✔
b. Infection will resolve only with a combination of antituberculous drugs, and then it may take weeks
c. Infection is short lasting and self-resolving and requires no treatment
d. Infection could have been prevented by avoiding cat feces and undercooked or raw meat
e. If treated with antibiotics, the infection should resolve in 3-6 days
1632. Five days ago a 65-year-old woman with a lower urinary tract infection began taking ampicillin. She now
has a fever and severe diarrhea. Of the organisms listed, which one is MOST likely to be the cause of the
diarrhea?
a. Bacteroides fragilis
b. Proteus mirabilis
c. Bordetella pertussis
d. Clostridium difficile ✔
1633. Children at day care centers in the United States have a high rate of infection with which one of the
following?
a. Ascaris lumbricoides
b. Enterobius vermicularis ✔
c. Entamoeba histolytica
d. Necator americanus
1634. A 32-year-old man who recently visited Nepal on business presents to the emergency department with
profuse watery diarrhea flecked with mucus, and severe dehydration. Which of the following correctly
describes the causal agent?
a. Gram-negative curved rod; toxin that increases cAMP
b. Gram-negative rod; toxin that inhibits protein synthesis
c. Gram-negative rod; toxin that increases cAMP ✔
d. Intoxication with a heat labile toxin that blocks the release of acetylcholine
e. Gram-negative curved rod; toxin that inhibits protein synthesis
1635. Each of the following statements concerning Diphyllobothrium latum is correct EXCEPT:
a. D. latum has operculated eggs.
b. D. latum causes a megaloblastic anemia due to vitamin B12 deficiency.
c. D. latum is a tapeworm that has a scolex with a circle of hooks. 11
d. D. latum is transmitted by undercooked fish.
1636. Each of the following statements concerning gram-negative rods is correct EXCEPT:
a. E. coli ferments lactose, whereas the enteric pathogens Shigella and Salmonella do not.
b. Escherichia coli is part of the normal flora of the colon; therefore, it does not cause diarrhea. ✔
c. Klebsiella pneumoniae, although a cause of pneumonia, is part of the normal flora of the colon.
d. Proteus species are highly motile organisms that are found in the human colon and cause urinary tract
infections.
1637. Each of the following parasites passes through the lung during human infection EXCEPT:
a. Wuchereria bancrofti ✔
b. Ascaris lumbricoides
c. Necator americanus
d. Strongyloides stercoralis
1638. Pigs or dogs are the source of human infection by each of the following parasites EXCEPT:
a. Trichinella spiralis
b. Ascaris lumbricoides ✔
c. Echinococcus granulosus
d. Taenia solium
1639. Endotoxin of which gram negative bacteria have no part in pathogenesis of disease:
a. Pseudomonas aeruginosa
b. E. coli
c. Klebsiella
d. Vibrio cholera ✔
1640. In the U.S., a baby has the greatest chance of acquiring which virus in utero?
a. Rubella virus
b. Hepatitis B virus
c. Herpes simplex virus
d. Respiratory syncytial virus
e. Cytomegalovirus ✔
1641. Which one of the following is the BEST explanation for the selective action of acyclovir (acycloguanosine)
in herpes simplex virus (HSV)-infected cells?
a. Acyclovir binds specifically to viral receptors only on the surface of the HSV-infected cell.
b. Acyclovir selectively inhibits the RNA polymerase in the HSV virion.
c. Acyclovir specifically blocks the matrix protein of HSV, thereby preventing release of progeny HSV.
d. Acyclovir is phosphorylated by a virus-encoded phosphokinase only within HSV-infected cells. ✔
1642. Varicella-zoster virus and herpes simplex virus share many characteristics. Which one of the following
characteristics is NOT shared?
a. Linear, double-stranded DNA genome
b. Inapparent disease, manifested only by virus shedding, is common ✔
c. Vesicular rash
d. Persistence of latent virus after recovery from acute disease
1643. A 24-year-old primiparous woman in her eighth month of gestation develops a positive IgM titer to
Toxoplasma gondii for the first time. She should be advised by her physician that
a. a newborn with a positive anti-Toxoplasma IgG response should be treated with anti-parasitics
b. major organ damage can be reversed by prompt treatment of the newborn
c. this child and all future fetuses are likely to be infected
d. retinochoroiditis can be prevented by drug treatment of an infant with a positive IgM response ✔
e. future infections can be avoided by proper vaccination and worming of cats
1644. Which one of the following outcomes is MOST common following a primary herpes simplex virus
infection?
a. Persistent asymptomatic viremia
b. Persistent cytopathic effect in infected cells
c. Complete eradication of virus and virus-infected cells
d. Establishment of latent infection ✔
1645. Which one of the following statements about genital herpes is LEAST accurate?
a. Initial disease episodes are generally more severe than recurrent episodes.
b. Multinucleated giant cells with intranuclear inclusions are found in the lesions.
c. Genital herpes can be transmitted in the absence of apparent lesions.
d. Acyclovir reduces the number of recurrent disease episodes by eradicating latently infected cells. ✔
1646. Each of the following statements concerning Toxoplasma gondii is correct EXCEPT:
a. T. gondii can be transmitted across the placenta to the fetus.
b. T. gondii can be transmitted by cat feces.
c. T. gondii can cause encephalitis in immunocompromised patients.
d. T. gondii can be diagnosed by finding trophozoites in the stool. ✔
1647. Herpes simplex virus type 1 (HSV-1) is distinct from HSV-2 in several different ways. Which one of the
following is the LEAST accurate statement?
a. HSV-1 causes lesions above the umbilicus more frequently than HSV-2 does.
b. Infection by HSV-1 is not associated with any tumors in humans.
c. Antiserum to HSV-1 neutralizes HSV-1 much more effectively than HSV-2.
d. HSV-1 causes frequent recurrences, whereas HSV-2 infection rarely recurs. ✔
1648. Herpes simplex virus and cytomegalovirus share many features. Which one of the following features is
LEAST likely to be shared?
a. Important cause of morbidity and mortality in the newborn
b. Important cause of serious disease in immunosuppressed individuals
c. Congenital abnormalities due to transplacental passage ✔
d. Mild or inapparent infection
1649. A 6-year-old girl presents to the emergency department with a fever and a lacy body rash. Her mother says
that yesterday the rash was only on her face, but by this morning, had spread to her trunk and extremities.
Which of the following agents is most likely?
a. Rubella
b. Varicella zoster virus
c. Parvovirus B19 ✔
d. Measles
e. HHV-6
1650. The neuraminidase of influenza virus exhibit all the following properties EXCEPT
a. lowers the viscosity of the mucous film in the respiratory tract
b. forms a spike structure composed of four identical monomers, each with enzyme activity
c. facilitates release of virus particles from infected cells
d. is embedded in the outer surface of the viral envelope
e. attaches with the sialic acid receptor present in upper respiratory tract ✔
1651. Internationally accepted naming convention is used for the nomenclature of Influenza Virus. WHO has
reported the outbreak of A/duck/X/Y/Z (H1N1) influenza virus? What does X denotes in this
nomenclature?
a. Strain Id
b. Year of isolation
c. Origin ✔
d. Type
1652. Which of the following statements concerning antigenic drift in influenza viruses is correct?
a. It results in major antigenic changes
b. It results in new sub types over time ✔
c. It is due to frame shift mutations in viral genes
d. It is exhibited by only influenza A viruses
e. It affects predominantly the matrix protein
1653. A 42 year old man developed a “flu-like” syndrome with fever, sore throat, headache, and myalgia. Viral
culture was ordered for laboratory confirmation of influenza. Which of the following would be the best
specimen for isolating virus responsible for the infection?
a. Blood
b. Vesicle fluid
c. Stool
d. Saliva
e. Nasopharyngeal washing ✔
1654. All of the following is responsible for the genetic diversity of Influenza A virus EXCEPT
a. RNA Virus
b. Ability to grow in many species
c. Ability to attach in sialic acid receptor ✔
d. Segmented genome
1655. What virus commonly undergoes both antigenic shifts and antigenic drifts?
a. Influenza ✔
b. HIV
c. Rabies
d. Rotavirus
e. Ebola
1656. The type specific antigen (A, B or C) of influenza viruses is found on which viral constituent?
a. Hemagglutinin
b. Lipid in viral envelope
c. Major structural protein
d. Nucleocapsid ✔
e. Neuraminidase
1657. Which of the following statements reflects the pathogenesis of influenza virus?
a. Pneumonia is not associated with secondary bacterial infections
b. The virus frequently establishes persistent infections in the lung
c. The virus enters the host in airborne droplets ✔
d. Viral infection does not kill cells in the respiratory tract
e. Viremia is common
1658. Influenza virus contains hemagglutinin and neuraminidase. What does these represent?
a. Proteins found in the nucleus of influenza virus
b. Proteins that surround each segment of the nucleic acid in influenza
c. Glycoproteins on influenza virus that contribute to virulence ✔
d. Glycoprotein receptors on influenza’s target cells
e. Exotoxins produced by the influenza virus
1659. All of the following statements correctly describes the properties of Influenza A virus EXCEPT
a. Single stranded positive sense RNA virus ✔
b. Viral transcription occurs in the nucleus
c. Mature virion are released by budding
d. Attaches to cell-surface sialic acid
1660. Which of the following items could be sterilized by dry heat sterilization?
a. glass pipette ✔
b. Intravenous (IV) solution
c. Plastic IV bags
d. plastic petri dishes
e. rubber gloves
1661. Which factor has an impact on the effectiveness of a disinfectant?
a. the disinfectant solution is more potent after several uses
b. using soap in the sanitization process is desirable
c. items should be placed in the disinfectant bath wet
d. evaporation can alter the chemical makeup of the solution ✔
1662. Which of the following substances can sterilize?
a. Chlorine
b. Alcohol
c. Cetylpyridinium chloride
d. Silver nitrate
e. Ethylene oxide ✔
1663. Which of the following process does not kill bacterial endospores?
a. Autoclave
b. Incineration
c. Pasteurization ✔
d. Hot air sterilization
1664. The most suitable way of sterilization of oils and fats is:
a. Pasteurization
b. Autoclaving
c. Hot air oven ✔
d. Tyndallisation
1665. Any process that destroys the non-spore-forming contaminants on inanimate objects is:
a. antisepsis
b. Disinfection ✔
c. degermation
d. sterilization
1666. Which is a form of cold sterilization
a. Infrared rays
b. Gamma rays ✔
c. UV rays
d. Steam sterilization
1667. Browne’s tube is used as indicator for efficacy of:
a. Ultraviolet rays
b. Filtration
c. Heat sterilization ✔
d. Chemical sterilization
1668. Identify the term that is used to ensure surgical instruments are free from micro-organisms:
a. Debrided
b. Disinfected
c. Sterilization ✔
d. Cleaned
1669. All of the following are sporicidal except
a. glutaraldehyde
b. alcohol ✔
c. formaldehyde
d. stabilized hydrogen peroxide
e. ethylene oxide
1670. The lowest temperature that kills all microorganisms in a liquid suspension in 10 minutes is known as the
a. thermal death time
b. thermal death temperature
c. thermal death point ✔
d. decimal reduction time
e. D value
1671. The time in minutes at a specific temperature needed to kill a population of cells is the
a. D value
b. thermal death temperature
c. decimal reduction time
d. F value ✔
e. thermal death point
1672. Which of the following is commonly used in the laboratory setting to provide a measure of protection
against transmission of the human immunodeficiency virus (HIV)?
a. germicidal soap
b. bleach
c. hot air
d. iodine ✔
1673. Which of the following method is best to sterilize heat labile solutions?
a. Autoclave
b. Hot Air Oven
c. Membrane filtration ✔
d. Pasteurization
1674. Which of the following scientist introduced the “Sterilization” technique?
a. Louis Pasteur ✔
b. Joseph Lister
c. John Needham
d. Robert Koch
1675. Sharp instruments should not be sterilized by:
a. Autoclaving
b. Hot air ovens ✔
c. Boiling
d. Chemical disinfectants
1676. Which of the following methods of infection control requires the use of an autoclave?
a. irradiation
b. sanitization
c. disinfection
d. sterilization ✔
1677. Which of the following is permissible in ultrasonic cleaning?
a. place all instruments together in the special bath
b. remove the instruments from the ultrasonic cleaner and immediately wrap in a towel without rinsing
c. place all instruments with hinges or ratchets in the ultrasonic cleaner in the closed position
d. reuse the ultrasonic cleaning solution for several baths ✔
1678. Which of the following methods is useful for sterilization of antisera?
a. Autoclaving
b. Tyndallisation
c. Hot air oven
d. Filtration ✔
1679. Tyndallization is a type of
a. Pasteurization
b. Boiling
c. Intermittent sterilization ✔
d. Dry heat
1680. Which of the following disinfectants acts by disrupting microbial membranes?
a. Aldehydes
b. Sterilizing gases
c. Heavy metals
d. Halogens
e. Cationic detergents ✔
1681. Which of the following is most effective for sterilizing mattresses and plastic Petri plates?
a. Glutaraldehyde
b. Chlorine
c. Ultraviolet radiation
d. Ethylene oxide ✔
e. Autoclave
1682. Each of the following statements concerning the killing of bacteria is correct EXCEPT:
a. Iodine kills by causing the formation of thymine dimers in bacterial DNA.
b. An autoclave uses steam under pressure to reach the killing temperature of 121°C.
c. A 70% solution of ethanol kills more effectively than absolute (100%) ethanol. ✔
d. The pasteurization of milk kills pathogens but allows many organisms and spores to survive.
1683. Which of the following is bactericidal?
a. Membrane filtration
b. Ionizing radiation ✔
c. Freeze-drying
d. Deep freezing
1684. Identify the term that can describe a disinfectant that can inhibit the growth of fungi:
a. fungistatic
b. microbiostatic
c. microbicidal
d. fungicidal 11
1685. A 13-year-old boy from India was brought to the emergency room with a prolapsed rectum. Examination
of the rectum reveals small worms that resemble whips attached to the mucosa. A stool sample reveals eggs
that are barrel shaped, with bipolar plugs. Which of the following is the most likely cause?
a. Trichuris trichiura ✔
b. Echinococcus granulosus
c. Ascaris lumbricoides
d. Giardia lamblia
e. Enterobius vermicularis
f. Entamoeba histolytica
1686. Pigs or dogs are the source of human infection by each of the following parasites EXCEPT:
a. Echinococcus granulosus
b. Ascaris lumbricoides ✔
c. Trichinella spiralis
d. Taenia solium
1687. Each of the following statements concerning hookworm infection is correct EXCEPT:
a. Hookworm infection is acquired by humans when filariform larvae penetrate the skin.
b. Hookworm infection can be diagnosed by finding the trophozoite in the stool. ✔
c. Hookworm infection is caused by Necator americanus.
d. Hookworm infection can cause anemia.
1688. Each of the following statements concerning trichinosis is correct EXCEPT:
a. Trichinosis is caused by a protozoan that has both a trophozoite and a cyst stage in its life cycle. ✔
b. Eosinophilia is a prominent finding.
c. Trichinosis is acquired by eating undercooked pork.
d. Trichinosis can be diagnosed by seeing cysts in muscle biopsy specimens.
1689. Each of the following parasites passes through the lung during human infection EXCEPT:
a. Strongyloides stercoralis
b. Necator americanus
c. Ascaris lumbricoides
d. Wuchereria bancrofti ✔
1690. Children at day care centers in the United States have a high rate of infection with which one of the
following?
a. Ascaris lumbricoides
b. Entamoeba histolytica
c. Enterobius vermicularis ✔
d. Necator americanus
1691. At a school nurse’s request, a clinic in rural South Carolina sees a 9-year-old girl who appears listless and
inattentive, although hearing and visual testing has been within normal limits. The physician finds the child
thin, with the “potbelly” of malnutrition, and orders a fecal exam and CBC. The CBC reveals a microcytic,
hypochrornic anemia, and the fecal exam detects brown, oval nematode eggs approximately 65 microns in
size, too numerous to count. What was the most likely means by which this child was infected?
a. Ingestion of larvae
b. Skin penetration by larvae ✔
c. Ingestion of ova
d. Ingestion of cysts in muscle
e. Mosquito transmission of sporozoites
1692. Each of the following parasites is transmitted by eating inadequately cooked fish or seafood EXCEPT:
a. Paragonimus westermani
b. Clonorchis sinensis
c. Diphyllobothrium latum
d. Ancylostoma duodenale ✔
1693. A 4-year-old boy presents to his pediatrician with intense perianal itching. His mother explains that the
child has also been extremely irritable during the day and has not been sleeping well at night. Eggs with a
flattened side were identified by the laboratory technician from a piece of scotch tape brought in by the
parent. Infection with which of the following organisms is most likely?
a. Echinococcus granulosus
b. Enterobius vermicularis ✔
c. Entamoeba histolytica
d. Trichuris trichiura
e. Ascaris lumbricoides
1694. The mother of a 4-year-old child notes that her child is sleeping poorly and scratching his anal area. You
suspect the child may have pinworms. Which one of the following is the BEST method to make that
diagnosis?
a. Determine the titer of IgE antibody against the organism
b. Examine a blood smear for the presence of microfilaria
c. Examine transparent adhesive tape for the presence of eggs ✔
d. Examine the stool for the presence of cysts
e. Examine the stool for the presence of trophozoites
1695. Each of the following statements concerning Ascaris lumbricoides is correct EXCEPT:
a. A. lumbricoides is one of the largest nematodes.
b. A. lumbricoides can cause pneumonia.
c. A. lumbricoides is transmitted by ingestion of eggs.
d. Both dogs and cats are intermediate hosts of A. lumbricoides. ✔
1696. Each of the following statements concerning Strongyloides stercoralis is correct EXCEPT:
a. Migrating larvae of S. stercoralis induce a marked eosinophilia.
b. S. stercoralis is acquired by ingestion of eggs. ✔
c. S. stercoralis undergoes a free-living life cycle in soil.
d. S. stercoralis produces filariform larvae.
1697. Each of the following organisms is an important cause of urinary tract infections EXCEPT:
a. Proteus mirabilis
b. Escherichia coli
c. Klebsiella pneumoniae
d. Bacteroides fragilis ✔
1698. A 45-year-old woman presents to the emergency department with intense pain in her lower back and a
burning sensation upon urination. A urine culture was taken and plated on MacConkey agar. Gram-
negative rods that did not ferment lactose were identified. Which virulence factor of the causal agent is
most important to pathogenesis?
a. Coagulase
b. Catalase
c. Capsule
d. Exotoxin
e. Urease ✔
1699. A 65-year-old man develops dysuria and hematuria. A Gram stain of a urine sample shows gram-negative
rods. Culture of the urine on EMB agar reveals lactose-negative colonies without evidence of swarming
motility. Which one of the following organisms is MOST likely to be the cause of his urinary tract
infection?
a. Pseudomonas aeruginosa ✔
b. Enterococcus faecalis
c. Escherichia coli
d. Proteus vulgaris
1700. A 9-year-old child develops glomerulonephritis a week after she was treated for a sore throat. The causal
agent is identified by serotyping of the
a. capsule
b. teichoic acids
c. M proteins ✔
d. outer membrane proteins
e. pili
1701. A 4-year-old girl has papular and pustular lesions on her face. The lesions are exuding a honey-colored
serous fluid. You make a clinical diagnosis of impetigo. A Gram stain of the exudate reveals gram-positive
cocci in chains, and a culture reveals ß-hemolytic colonies on blood agar. For which one of the following
sequelae is she MOST at risk?
a. Bloody diarrhea
b. Paralysis of the facial nerve (Bell’s palsy)
c. Red blood cells and albumin in her urine ✔
d. Blurred vision
1702. A 70-year-old man is found to have a hard mass in his prostate, which is suspected to be a carcinoma.
Twenty-four hours after surgical removal of the mass, he develops fever to 39°C and has several shaking
chills. Of the organisms listed, which one is LEAST likely to be involved?
a. Legionella pneumophila ✔
b. Escherichia coli
c. Enterococcus faecalis
d. Klebsiella pneumoniae
1703. Acute glomerulonephritis is a nonsuppurative complication that follows infection by which one of the
following organisms?
a. Streptococcus pyogenes ✔
b. Enterococcus faecalis
c. Streptococcus agalactiae
d. Streptococcus pneumoniae
1704. A 60-year-old woman is hospitalized following a stroke and develops a highgrade fever with chills. She is
catheterized due to urinary incontinence and receives cephalosporin for treatment of pneumonia. Blood
cultures and Gram stain are performed by the laboratory. The organisms isolated are gram-positive cocci
that are catalase-negative and capable of growth in 6.5% sodium chloride. Which of the following is the
most likely causal agent?
a. Staphylococcus aureus
b. Enterococcus faecalis ✔
c. Viridans streptococci
d. Streptococcus pyogenes
e. Staphylococcus epidermidis
1705. A 27-year-old woman, after returning home from her honeymoon, has developed urinary frequency,
dysuria, and urgency. Her urine is grossly bloody. Which lab data are most likely to define the causal
agent?
a. A gram-positive bacillus grown on a low oxidation-reduction medium
b. An optochin-resistant, catalase-negative, gram-positive coccus
c. A gram-negative diplococcus, which is oxidase positive but does not ferment maltose
d. A gram-positive coccus, which is catalase positive and coagulase negative
e. A gram-negative bacterium capable of reducing nitrates to nitrites ✔
1706. Four weeks after his arrival from Africa, a 24-year-old graduate student presents with blood in his urine.
Microscopic examination of his urine reveals the presence of eggs with terminal spines. In the interview he
admits that he has been working on his family’s rice field occasionally since his early childhood. The most
likely etiologic agent of his complaint is
a. Schistosoma haematobium ✔
b. Schistosoma japonicum
c. Entamoeba histolytica
d. Schistosoma mansoni
e. Fasciolopsis buski
1707. The first vaccine for human use produced using recombinant DNA technology was the:
a. Hepatitis A vaccine
b. Polio vaccine
c. AIDS vaccine
d. MMR vaccine
e. Hepatitis B vaccine ✔
1708. Disease caused by which one of the following bacteria can be prevented by a toxoid vaccine?
a. Haemophilus influenzae
b. Streptococcus pneumoniae
c. Corynebacterium diphtheriae ✔
d. Salmonella typhi
e. Neisseria meningitidis
1709. The two most common types of viral vaccines are killed vaccines and live, attenuated vaccines. Regarding
these vaccines, which one of the following statements is the MOST accurate?
a. Killed vaccines induce a longer-lasting response than do live, attenuated vaccines.
b. Killed vaccines induce a broader range of immune responses than do live, attenuated vaccines.
c. Killed vaccines are safer to give to immunocompromised patients than are live, attenuated vaccines. ✔
d. Killed vaccines are no longer used in this country because they do not induce secretory IgA.
1710. A 45-year-old man was attacked by a bobcat and bitten repeatedly about the face and neck. The animal
was shot by a companion and brought back to the public health authorities. Once you decide to immunize
against rabies virus, how would you proceed?
a. Use hyperimmune serum and active immunization ✔
b. Use active immunization only
c. Use hyperimmune serum only
1711. A living microbe with reduced virulence that is used for vaccination is considered:
a. Denatured
b. Dormant
c. Attenuated ✔
d. A toxoid
e. Virulent
1712. To design a vaccine against HIV infection, a logical goal would be to alter some native molecule or product
of the virion in order to make it highly immunogenic. If you wished to prevent the attachment of the virus
to helper T lymphocytes, which molecule or family of molecules might best be targeted?
a. gp41
b. p24
c. nucleocapsid protein
d. gp l20 11
e. p 17
1713. Each of the following statements concerning rubella vaccine is correct EXCEPT:
a. The immunogen in the vaccine is killed rubella virus. ✔
b. The vaccine prevents reinfection, thereby limiting the spread of virulent virus.
c. The incidence of both childhood rubella and congenital rubella syndrome has decreased significantly
since the advent of the vaccine.
d. The vaccine induces antibodies that prevent dissemination of the virus by neutralizing it during the
viremic stage.
1714. Which one of the following statements concerning immunization against diseases caused by clostridia is
CORRECT?
a. Antitoxin against tetanus protects against botulism as well, because the two toxins share antigenic sites.
b. Immunization with tetanus toxoid induces effective protection against tetanus toxin. ✔
c. The toxoid vaccine against Clostridium difficile infection should be administered to
immunocompromised patients.
d. Vaccines containing alpha toxin (lecithinase) are effective in protecting against gas gangrene.
1715. The MOST important protective function of the antibody stimulated by tetanus immunization is:
a. To neutralize the toxin of the pathogen ✔
b. To prevent adherence of the pathogen
c. To prevent growth of the pathogen
d. To opsonize the pathogen (Clostridium tetani)
1716. Each of the following statements concerning the rabies vaccine for use in humans is correct EXCEPT:
a. If your patient is bitten by a wild animal (e.g., a skunk) the rabies vaccine should be given.
b. The virus in the vaccine is grown in human cell cultures, thus decreasing the risk of allergic
encephalomyelitis.
c. When the vaccine is used for postexposure prophylaxis, rabies immune globulin should also be given.
d. The vaccine contains live, attenuated rabies virus. 11
1717. Each of the following parasites has an intermediate host as part of its life cycle EXCEPT:
a. Echinococcus granulosus
b. Taenia solium
c. Trichomonas vaginalis ✔
d. Toxoplasma gondii
1718. A 24-year-old primiparous woman in her eighth month of gestation develops a positive IgM titer to
Toxoplasma gondii for the first time. She should be advised by her physician that
a. future infections can be avoided by proper vaccination and worming of cats
b. this child and all future fetuses are likely to be infected
c. a newborn with a positive anti-Toxoplasma IgG response should be treated with anti-parasitics
d. major organ damage can be reversed by prompt treatment of the newborn
e. retinochoroiditis can be prevented by drug treatment of an infant with a positive IgM response ✔
1719. Which of the following statement(s) regarding Plasmodium falciparum are true?
a. causes benign tertian malaria
b. is the only malarial parasite causing greater than 20% parasitaemia ✔
c. is associated with recurrent relapses after initial treatment because of liver hypnozoites
d. is the only cause of cerebral malaria
1720. A 35-year-old man comes to the emergency department complaining of high fever, chills, severe headache,
and confusion. He has recently returned from Africa. A peripheral blood smear reveals multiple ring
structures and crescent-shaped gametes. Which of the following organisms is the most likely cause?
a. Plasmodium ovale
b. Leishmania species
c. Plasmodium falciparum ✔
d. Plasmodium malariae
e. Plasmodium vivax
1721. After sporozoite gain entrance to human body it undergoes developmental cycle first in liver than in RBC,
only after which fever is seen. This incubation period varies between plasmodium species, and …………..
species has longest incubation period.
a. P. falciparum
b. P. vivax
c. P. ovale
d. P. malariae ✔
1722. After one week vacationing in Mexico, a 14-year-old girl presents with abdominal pain, nausea, bloody
diarrhea, and fever. Stool specimens are collected and sent to the laboratory for bacteriologic and
parasitologic examination. Bacterial cultures are negative for intestinal pathogens. The laboratory report
reveals organisms with red blood cells inside them. The most likely causal agent is
a. Cryptosporidium parvum
b. Giardia lamblia
c. Shigella dysenteriae
d. Entamoeba histolytica ✔
e. Toxoplasma gondii
1723. Which one of the following protozoa primarily infects macrophages?
a. Trypanosoma cruzi
b. Leishmania donovani ✔
c. Plasmodium vivax
d. Trichomonas vaginalis
1724. Trophozoites, Schizonts and gametocytes of all the malarial parasites are seen in the peripheral blood
smear except;
a. P. falciparum ✔
b. P. ovale
c. P. vivax
d. P. malariae
1725. How many pairs of flagella are present in the trophozoite of Giardia lamblia?
a. Two pairs
b. Three pairs
c. Four pairs ✔
d. One pair
1726. Each of the following statements concerning mucormycosis is correct EXCEPT:
a. Hyphae typically invade blood vessels and cause necrosis of tissue.
b. The fungi that cause mucormycosis are transmitted by airborne asexual spores.
c. Ketoacidosis in diabetic patients is a predisposing factor to mucormycosis.
d. Tissue sections from a patient with mucormycosis show budding yeasts. ✔
1727. Several fungi are associated with disease in immunocompromised patients. Which one of the following is
the LEAST frequently associated?
a. Mucor species
b. Aspergillus fumigatus
c. Malassezia furfur ✔
d. Cryptococcus neoformans
1728. A 33-year-old HIV-positive man complains of headache and blurred vision. Physical examination reveals
papilledema and ataxia. A head CT scan is normal, but CSF obtained by lumbar puncture reveals
encapsulated organisms visible by India ink. Which of the following is true concerning this organism?
a. It exists as a mycelial form at room temperature and a yeast at 37°C
b. It is an encapsulated non dimorphic yeast found worldwide ✔
c. It consists of branching septate hyphae
d. It is a nonencapsulated dimorphic yeast that reproduces by budding
e. It can also be seen as “spaghetti and meatballs” on KOH stain
1729. You have made a clinical diagnosis of meningitis in a 50-year-old immunocompromised woman. A latex
agglutination test on the spinal fluid for capsular polysaccharide antigen is positive. Of the following
organisms, which one is the MOST likely cause?
a. Candida albicans
b. Aspergillus fumigatus
c. Cryptococcus neoformans ✔
d. Histoplasma capsulatum
1730. Each of the following statements concerning Candida albicans is correct EXCEPT:
a. C. albicans causes thrush.
b. C. albicans is transmitted primarily by respiratory aerosol. ✔
c. Impaired cell-mediated immunity is an important predisposing factor to disease.
d. C. albicans is a budding yeast that forms pseudohyphae when it invades tissue.
1731. You suspect that your patient’s disease may be caused by Cryptococcus neoformans. Which one of the
following findings would be MOST useful in establishing the diagnosis?
a. The finding of encapsulated budding cells in spinal fluid ✔
b. A positive heterophil agglutination test for the presence of antigen
c. Recovery of an acid-fast organism from the patient’s sputum
d. A history of recent travel in the Mississippi River valley area
1732. A 55-year-old man who recently recovered uneventfully from a heart valve transplant presents to the
emergency room with pleuritic chest pain, hemoptysis, fever, and chills. While he is being examined, he has
a myocardial infarction and the medical team is unable to revive him. An autopsy revealed septate, acutely
branching hyphae in many tissues. Which of the following organisms is most likely to be identified?
a. Aspergillus fumigatus ✔
b. Mucor species
c. Histoplasma capsulatum
d. Cryptococcus neoformans
e. Blastomyces dermatitidis
1733. Your patient is a woman with a vaginal discharge. You suspect, on clinical grounds, that it may be due to
Candida albicans. Which one of the following statements is LEAST accurate or appropriate?
a. The clinical laboratory can use germ tube formation to identify the isolate as C. albicans.
b. Antibiotics predispose to Candida vaginitis by killing the normal flora lactobacilli that keep the vaginal
pH low.
c. Culture of the discharge on Sabouraud’s agar should produce a white mycelium with aerial conidia. ✔
d. A Gram stain of the discharge should reveal budding yeasts.
1734. Aspergillus fumigatus can be involved in a variety of clinical conditions. Which one of the following is
LEAST likely to occur?
a. Allergy following inhalation of airborne particles of the fungus
b. Colonization of tuberculous cavities in the lung
c. Tissue invasion in immunocompromised host
d. Thrush ✔
1735. A 42-year-old man who has AIDS presents to his physician with progressively increasing dyspnea over the
past 3 weeks. He also complains of a dry, painful cough, fatigue, and low-grade fever. A chest x-ray reveals
bilateral symmetrical interstitial and alveolar infiltration. Which of the following agents is the most likely
cause of the above?
a. Toxoplasma gondii
b. Histoplasma capsulatum
c. Pneumocystis jiroveci ✔
d. Cryptococcus neoformans
e. Cryptosporidium parvum
1736. A 1 5-year-old member of the high school swim team notices painless, umbilicated cutaneous lesions on the
toes. Large eosinophilic cytoplasmic inclusions are present in the affected epithelia. What is the most likely
causal agent?
a. Adenovirus
b. Herpes simplex virus
c. Human papilloma virus
d. Molluscum contagiosum virus ✔
1737. Nagler’s reaction is useful for the identification of
a. Clostridium botulinum
b. Clostridium perfringens ✔
c. Clostridium tetani
d. Clostridium difficile
1738. A 35-year-old man who is positive for HIV develops sepsis with the subsequent development of a necrotic
lesion on the buttock that has a black center and an erythematous margin. Which of the following is the
most likely causal agent?
a. Enterococcus faecalis ✔
b. Pseudomonas aeruginosa
c. Bacillus anthracis
d. Clostridium perfringens
1739. Each of the following statements concerning Clostridium perfringens is correct EXCEPT:
a. It causes gas gangrene.
b. It produces an exotoxin that degrades lecithin and causes necrosis and hemolysis.
c. It is a gram-negative rod that does not ferment lactose. ✔
d. It causes food poisoning.
1740. A 4-year-old girl has papular and pustular lesions on her face. The lesions are exuding a honey-colored
serous fluid. You make a clinical diagnosis of impetigo. A Gram stain of the exudate reveals gram-positive
cocci in chains, and a culture reveals ß-hemolytic colonies on blood agar. For which one of the following
sequelae is she MOST at risk?
a. Blurred vision
b. Bloody diarrhea
c. Red blood cells and albumin in her urine ✔
d. Paralysis of the facial nerve (Bell’s palsy)
1741. Each of the following statements concerning clostridia is correct EXCEPT:
a. Anaerobic conditions at the wound site are not required to cause tetanus, because spores will form in the
presence of oxygen. ✔
b. Pathogenic clostridia are found both in the soil and in the normal flora of the colon.
c. Botulism, which is caused by ingesting preformed toxin, can be prevented by boiling food prior to eating.
d. Antibiotic-associated (pseudomembranous) colitis is due to a toxin produced by Clostridium difficile.
1742. Contamination of umbilical cord at birth may lead to infection by:
a. Clostridium difficile
b. Clostridium perfringens
c. Clostridium botulinum
d. Clostridium tetani ✔
1743. Stormy clot reaction is useful in the identification of:
a. Clostridium tetani
b. Clostridium perfringens ✔
c. Clostridium difficile
d. Clostridium botulinum
1744. Each of the following statements concerning Staphylococcus aureus is correct EXCEPT:
a. The coagulase test is positive.
b. Endotoxin is an important pathogenetic factor. ✔
c. Treatment should include a ß-lactamase–resistant penicillin.
d. Gram-positive cocci in grapelike clusters are seen on Gram-stained smear.
1745. A 25-year-old man gets into a fight at the local bar and punches another patron in the mouth. The
following day his fist becomes infected and he visits a local urgent care center. Exudate from the wound is
cultured on blood and chocolate agar and reveals gram-negative rods that have a bleach-like odor. Which
of the following agents is the most likely cause?
a. Actinobacillus actinomycetemcomitans
b. Pseudomonas aeruginosa
c. Eikenella corrodens ✔
d. Kingella kingae
e. Cardiobacterium hominis
1746. A 50-year-old homeless alcoholic has a fever and is coughing up 1 cup of green, foul-smelling sputum per
day. You suspect that he may have a lung abscess. Which one of the following pairs of organisms is MOST
likely to be the cause?
a. Fusobacterium nucleatum and Peptostreptococcus intermedius
b. Clostridium perfringens and Chlamydia psittaci
c. Listeria monocytogenes and Legionella pneumophila
d. Nocardia asteroides and Mycoplasma pneumoniae ✔
1747. Which one of the following organisms is MOST likely to be the cause of pneumonia in an
immunocompetent young adult?
a. Nocardia asteroides
b. Mycoplasma pneumoniae ✔
c. Serratia marcescens
d. Legionella pneumophila
1748. Each of the following statements concerning Corynebacterium diphtheriae is correct EXCEPT:
a. Toxin production is dependent on the organism’s being lysogenized by a bacteriophage.
b. Antitoxin should be used to treat patients with diphtheria.
c. Diphtheria toxoid should not be given to children younger than 3 years because the incidence of
complications is too high. ✔
d. C. diphtheriae is a gram-positive rod that does not form spores.
1749. For which one of the following bacterial vaccines are toxic side effects an important concern?
a. The vaccine containing pneumococcal polysaccharide
b. The vaccine containing tetanus toxoid
c. The vaccine containing killed Bordetella pertussis ✔
d. The vaccine containing diphtheria toxoid
1750. Each of the following statements concerning certain gram-negative rods is correct EXCEPT:
a. Legionella pneumophila infection is acquired by inhalation of aerosols from environmental water sources.
b. Whooping cough, which is caused by Bordetella pertussis, is on the rise because changing antigenicity of
the organism has made the vaccine relatively ineffective. ✔
c. In unimmunized individuals, invasive disease caused by Haemophilus influenzae is most often due to
strains possessing a type b polysaccharide capsule.
d. Pseudomonas aeruginosa causes wound infections that are characterized by blue-green pus as a result of
pyocyanin production.
1751. A bone marrow transplant recipient becomes febrile and hypoxic and chest films demonstrate diffuse
interstitial pneumonia. What is the most likely causal agent?
a. Cytomegalovirus ✔
b. Varicella-zoster virus
c. Molluscum contagiosum virus
d. Herpes simplex virus
e. Paramyxovirus
1752. A 1 9-year-old migrant worker from the southwestern U.S. is brought to the family doctor complaining of
cough, pleuritic chest pain, fever, and malaise. He also complains of a backache and headache. He is found
to have an erythematous skin rash on his lower limbs. A chest radiograph reveals several calcifying lesions.
Which of the following structures is most likely to be found?
a. Septate hyphae branching dichotomously at acute angles
b. Spherules with endospores ✔
c. Monomorphic encapsulated yeast
d. Nonseptate hyphae with broad angles
e. Broad-based budding yeast
1753. A 42-year-old man who has AIDS presents to his physician with progressively increasing dyspnea over the
past 3 weeks. He also complains of a dry, painful cough, fatigue, and low-grade fever. A chest x-ray reveals
bilateral symmetrical interstitial and alveolar infiltration. Which of the following agents is the most likely
cause of the above?
a. Cryptococcus neoformans
b. Toxoplasma gondii
c. Cryptosporidium parvum
d. Histoplasma capsulatum
e. Pneumocystis jiroveci ✔
1754. A 16-year-old has pneumonia with a dry, hacking cough. The x-ray pattern shows a light, diffuse
infiltrative pattern. The most likely organism producing these symptoms is
a. A coagulase positive, gram-positive, catalase positive coccus in clusters
b. A non-Gram-staining bacterium requiring sterols ✔
c. A bacillus showing granules when stained with methylene blue
d. A gram-positive bacillus grown on a low oxidation-reduction medium
e. A bacitracin-sensitive, catalase-negative gram-positive coccus
1755. The MOST important contribution of the capsule of Streptococcus pneumoniae to virulence is:
a. To accelerate tissue invasion by its collagenase-like activity
b. To retard phagocytosis by polymorphonuclear leukocytes ✔
c. To inhibit polymorphonuclear leukocyte chemotaxis
d. To prevent dehydration of the organisms on mucosal surfaces
1756. Which genetic material is found in pathogenic Corynebacterium diphtheriae but not in nonpathogenic
normal flora diphtheroids?
a. An F factor
b. A diphthamide on eEF-2
c. An integrated temperate phage ✔
d. An episome
1757. A 22-year-old woman complains of a persistent nonproductive cough and a fever of 101°F that came on
slowly over the last 4 days. Physical examination reveals some rales in the left lung base. A patchy infiltrate
is seen on chest X-ray. She works as a secretary in a law office and has not traveled recently. She is not
immunocompromised and has not been hospitalized recently. A sample of her serum agglutinates red blood
cells at 4°C but not at 37°C. Which one of the following BEST describes the organism that is the MOST
likely cause of her disease?
a. An acid-fast rod that forms colonies within 7 days
b. A very small bacterium that has no cell wall ✔
c. A filamentous gram-positive rod that is weakly acid-fast
d. A gram-negative diplococcus with a large capsule
e. A spirochete that has never been grown on blood agar
1758. Each of the following statements concerning Mycobacterium tuberculosis is correct EXCEPT:
a. Some strains of M. tuberculosis isolated from patients exhibit multiple drug resistance (i.e., they are
resistant to both isoniazid and rifampin).
b. The antigen in the tuberculin skin test is a protein extracted from the organism.
c. M. tuberculosis grows slowly, often requiring 3 to 6 weeks before colonies appear.
d. M. tuberculosis contains a small amount of lipid in its cell wall and therefore stains poorly with the Gram
stain. ✔
1759. Your patient has a brain abscess that was detected 1 month after a dental extraction. Which one of the
following organisms is MOST likely to be involved?
a. Mycobacterium smegmatis
b. Lactobacillus acidophilus
c. Anaerobic streptococci ✔
d. Mycoplasma pneumoniae
1760. Each of the following statements concerning Pneumocystis carinii is correct EXCEPT:
a. P. carinii infections primarily involve the respiratory tract.
b. P. carinii symptomatic infections can be prevented by administering penicillin orally. ✔
c. P. carinii can be diagnosed by seeing cysts in tissue.
d. P. carinii infections are symptomatic primarily in immunocompromised patients.
1761. A 50-year-old homeless alcoholic has a fever and is coughing up 1 cup of green, foul-smelling sputum per
day. You suspect that he may have a lung abscess. Which one of the following pairs of organisms is MOST
likely to be the cause?
a. Listeria monocytogenes and Legionella pneumophila
b. Fusobacterium nucleatum and Peptostreptococcus intermedius
c. Clostridium perfringens and Chlamydia psittaci
d. Nocardia asteroides and Mycoplasma pneumoniae ✔
1762. A 22-year-old woman presents with “the worst sore throat I’ve ever had.” She also complains of fatigue
and anorexia. She is not immunocompromised and has not been hospitalized recently. On examination, she
is febrile to 38°C, the pharynx is inflamed, and there are a few tender cervical nodes bilaterally. There are
no white lesions on the tongue or pharynx. A throat culture grows alpha-hemolytic colonies on blood agar
that are optochin-resistant. Of the following, which one is the MOST likely cause?
a. Streptococcus pneumoniae
b. Epstein–Barr virus ✔
c. Streptococcus pyogenes
d. Parvovirus B19
e. Streptococcus mutans
1763. A 45-year-old man presents to the emergency department with shortness of breath and a productive cough.
His sputum was gelatinous and bloody. Gram stain of the sputum revealed numerous PMNs and gram-
negative rods. Which of the following descriptions is most likely to fit the patient?
a. IV drug user
b. Hiker
c. Homeless
d. Veterinarian
e. Alcoholic ✔
1764. Which one of the following organisms is MOST likely to be the cause of pneumonia in an
immunocompetent young adult?
a. Mycoplasma pneumoniae ✔
b. Nocardia asteroides
c. Serratia marcescens
d. Legionella pneumophila
1765. The MOST important way the host counteracts the function of the pneumococcal polysaccharide capsule is
via:
a. Activated macrophages
b. Anticapsular antibody ✔
c. Polysaccharide-degrading enzymes
d. T lymphocytes sensitized to polysaccharide antigens
1766. A 70-year-old man presents to the emergency department with a fever of 103.5°F, a dry cough, tachypnea,
and chest pain. History reveals he has been smoking since he was a teen. He mentions that several people at
the assisted living community where he resides have had similar symptoms. A sputum sample isolated
organisms that grew on buffered charcoal yeast extract agar and stained weakly gram-negative. Which of
the following properties is consistent with the above organism?
a. Optochin sensitive
b. Requires iron and cysteine for growth ✔
c. Capsule
d. No cell wall
e. Serpentine growth in vitro
1767. Which one of the following statements concerning Legionella pneumophila is CORRECT?
a. It is part of the normal flora of the colon.
b. It does not have a cell wall.
c. It causes atypical pneumonia, especially in those with reduced cell-mediated immunity. ✔
d. It cannot be grown on laboratory media.
1768. A 12-year-old boy presents to his pediatrician with fever, malaise, and a sore throat. Physical examination
reveals a fever of 103°F, cervical lymphadenopathy, and pharyngeal erythema. A swab is taken from some
of the tonsillar exudate and cultured on blood agar. Culture reveals beta hemolytic, gram-positive cocci,
and a rapid antigen test is positive. What is the major component that protects the causal agent from
osmotic damage?
a. Peptidoglycan ✔
b. Polysaccharide
c. Phospholipids
d. Teichoic acid
e. Lipopolysaccharide
1769. A 45-year-old homeless man presents to the emergency department with fever and night sweats, coughing
up blood. Acid-fast bacilli are identified in his sputum. Which of the following virulence factors allows the
causal agent to inhibit phagosome-lysosome fusion to survive intracellularly?
a. Sulfatides ✔
b. Calcium dipicolinate
c. Peptidoglycan
d. Tuberculin
e. Cord factor
1770. Each of the following statements concerning Mycobacterium tuberculosis is correct EXCEPT:
a. Some strains of M. tuberculosis isolated from patients exhibit multiple drug resistance (i.e., they are
resistant to both isoniazid and rifampin).
b. The antigen in the tuberculin skin test is a protein extracted from the organism.
c. M. tuberculosis contains a small amount of lipid in its cell wall and therefore stains poorly with the Gram
stain. ✔
d. M. tuberculosis grows slowly, often requiring 3 to 6 weeks before colonies appear.
1771. A 14-year-old girl develops a sore throat, fever, and earache of approximately 1 week duration. Upon
examination by her physician, an erythematous rash is noted covering most of her body and her tongue
appears bright red. Which of the following is the description of the causal agent?
a. Gram-positive coccus, alpha hemolytic, catalase negative
b. Gram-positive coccus, gamma hemolytic, catalase negative
c. Gram-positive coccus, beta hemolytic, catalase positive ✔
d. Gram-positive coccus, alpha hemolytic, catalase positive
e. Gram-positive coccus, beta hemolytic, catalase negative
1772. Each of the following statements concerning infection with Chlamydia psittaci is correct EXCEPT:
a. The organism appears purple in Gram-stained smears of sputum. ✔
b. The infection is more commonly acquired from a nonhuman source than from another human.
c. C. psittaci can be isolated by growth in cell culture and will not grow in blood agar.
d. The infection is more readily diagnosed by serologic tests than by isolation of the organism.
1773. A 22-year-old woman complains of a persistent nonproductive cough and a fever of 101°F that came on
slowly over the last 4 days. Physical examination reveals some rales in the left lung base. A patchy infiltrate
is seen on chest X-ray. She works as a secretary in a law office and has not traveled recently. She is not
immunocompromised and has not been hospitalized recently. A sample of her serum agglutinates red blood
cells at 4°C but not at 37°C. Which one of the following BEST describes the organism that is the MOST
likely cause of her disease?
a. A filamentous gram-positive rod that is weakly acid-fast
b. A spirochete that has never been grown on blood agar
c. An acid-fast rod that forms colonies within 7 days
d. A gram-negative diplococcus with a large capsule
e. A very small bacterium that has no cell wall ✔
1774. Which one of the following is NOT an important characteristic of Streptococcus pyogenes?
a. Beta-hemolysin
b. Polysaccharide group-specific substance
c. Protein A ✔
d. M protein
1775. Which genetic material is found in pathogenic Corynebacterium diphtheriae but not in nonpathogenic
normal flora diphtheroids?
a. A diphthamide on eEF-2
b. An episome
c. An integrated temperate phage ✔
d. Highly repetitive bacterial DNA
e. An F factor
1776. A 55-year-old woman had her rheumatic heart valve replaced with a prosthetic valve. Six blood cultures
became positive after 3 days of incubation. An optochin- resistant, catalase-negative gram-positive coccus
that was alpha-hemolytic was isolated. What was the most likely causal agent?
a. Pseudomonas aeruginosa
b. Streptococcus pneumoniae
c. Serratia marcescens
d. Streptococcus viridans ✔
e. Staphylococcus aureus
1777. Your patient has subacute bacterial endocarditis caused by a member of the viridans group of streptococci.
Which one of the following sites is MOST likely to be the source of the organism?
a. Colon
b. Skin
c. Urethra
d. Oropharynx ✔
1778. A 70-year-old woman is brought to the emergency department by her spouse with complaints of shortness
of breath and fever. Physical examination revealed a fever of 103°F, hypotension, and a diastolic murmur.
History revealed a cardiac valve replacement 5 years earlier. Three consecutive blood cultures taken
during febrile periods revealed gram-positive cocci that were catalase-positive and coagulase-negative.
Which of the following organisms is the most likely cause?
a. Enterococcus faecalis
b. Kingella kingae
c. Staphylococcus aureus
d. Staphylococcus epidermidis ✔
e. Staphylococcus saprophyticus
1779. Each of the following statements concerning kala-azar is correct EXCEPT:
a. Kala-azar is transmitted by the bite of sandflies.
b. Kala-azar is caused by Leishmania donovani.
c. Kala-azar can be diagnosed by finding amastigotes in bone marrow.
d. Kala-azar occurs primarily in rural Latin America. ✔
1780. Regarding Lyme disease, which one of the following is MOST accurate?
a. Mice are the main reservoir of the causative organism. ✔
b. The Lyme disease vaccine contains toxoid as the immunogen.
c. Fleas are the principal mode of transmission of the causative organism.
d. The causative organism is a small gram-positive rod.
e. The diagnosis in the clinical laboratory is typically made by culturing the organism on chocolate agar.
1781. Which one of the following protozoa primarily infects macrophages?
a. Trypanosoma cruzi
b. Leishmania donovani ✔
c. Trichomonas vaginalis
d. Plasmodium vivax
1782. A 35-year-old captain in the army reserves has been plagued by a painful, erosive lesion near his ear lobe
since his return from Operation Desert Storm several years ago. He denies exposure to the toxic by-
products of burning oil fields. Punch biopsy of the leading edge of the erosion reveals macrophages
distended with oval amastigotes. How was this infection acquired?
a. Bite of reduviid bug
b. Bite of tsetse fly
c. Contact with contaminated drinking water
d. Bite of Anopheles mosquito
e. Bite of sandfly ✔
f. Fecal contamination of food
1783. Listed below are five bacteria paired with a mode of transmission. Which one of the pairings is MOST
accurate?
a. Coxiella burnetii—bat guano
b. Borrelia burgdorferi—mosquito bite
c. Yersinia pestis—flea bite ✔
d. Rickettsia rickettsii—contaminated food
e. Haemophilus influenzae—penetrating wound contaminated with soil
1784. Each of the following statements concerning Toxoplasma gondii is correct EXCEPT:
a. T. gondii can be transmitted by cat feces.
b. T. gondii can be transmitted across the placenta to the fetus.
c. T. gondii can cause encephalitis in immunocompromised patients.
d. T. gondii can be diagnosed by finding trophozoites in the stool. ✔
1785. Each of the following statements concerning spirochetes is correct EXCEPT:
a. Species of Borrelia cause a tick-borne disease called relapsing fever.
b. Species of Treponema are part of the normal flora of the mouth.
c. The species of Leptospira that cause leptospirosis grow primarily in humans and are usually transmitted
by human-to-human contact. ✔
d. Species of Treponema cause syphilis and yaws.
1786. An outbreak of sepsis caused by Staphylococcus aureus has occurred in the newborn nursery. You are
called upon to investigate. According to your knowledge of the normal flora, what is the MOST likely
source of the organism?
a. Throat
b. Nose ✔
c. Vagina
d. Colon
1787. Which one of the following zoonotic illnesses has NO arthropod vector?
a. Lyme disease
b. Brucellosis ✔
c. Plague
d. Epidemic typhus
1788. A 81-year-old woman who is receiving chemotherapy for leukemia develops a fever to 40°C and has two
episodes of teeth-chattering chills, and her blood pressure drops to 80/20 mmHg. Of the following factors,
which one is MOST likely to be the cause of her fever, chills, and hypotension?
a. Coagulase
b. Lipid A ✔
c. Mycolic acid
d. Polysaccharide capsule
1789. A 68-year-old woman on chemotherapy for leukemia has developed sepsis due to an infection with
Escherichia coli . The following day the patient develops septic shock and dies. The structure on the
bacterium most likely responsible for causing septic shock in this patient is
a. capsule
b. teichoic acid
c. pili
d. spore
e. lipopolysaccharide ✔
1790. Each of the following statements concerning Trypanosoma cruzi is correct EXCEPT:
a. T. cruzi can be diagnosed by seeing amastigotes in a bone marrow aspirate.
b. T. cruzi typically affects heart muscle, leading to cardiac failure.
c. T. cruzi is transmitted by the reduviid bug.
d. T. cruzi occurs primarily in tropical Africa. ✔
1791. A 28-year-old woman with a previous history of rheumatic fever now has a fever for the past 2 weeks.
Physical examination reveals a new heart murmur. You suspect endocarditis and do a blood culture, which
grows a viridans group streptococcus later identified as Streptococcus sanguis. Of the following body sites,
which one is the MOST likely source of this organism?
a. Colon
b. Skin
c. Stomach
d. Vagina
e. Mouth ✔
1792. A 24-year-old primiparous woman in her eighth month of gestation develops a positive IgM titer to
Toxoplasma gondii for the first time. She should be advised by her physician that
a. this child and all future fetuses are likely to be infected
b. future infections can be avoided by proper vaccination and worming of cats
c. retinochoroiditis can be prevented by drug treatment of an infant with a positive IgM response ✔
d. major organ damage can be reversed by prompt treatment of the newborn
e. a newborn with a positive anti-Toxoplasma IgG response should be treated with anti-parasitics
1793. A 33-year-old man presents to the emergency department with a fever of 102.5°F, facial palsy, headache,
and malaise. A circular maculopapular rash was identified on the patients left shoulder; the patient was
unaware of the rash. The patient likely acquired the above infection via which of the following routes?
a. Respiratory route
b. Sexual contact
c. Consumption of contaminated food
d. Arthropod vector ✔
e. Direct contact with fomite
1794. Black water fever is a special manifestation of malaria caused by;
a. P. vivax
b. P. falciparum ✔
c. P. ovale
d. P. malariae
1795.
1796. Which one of the following statements concerning the organisms that cause brucellosis is CORRECT?
a. Brucellae are transmitted primarily by tick bite.
b. The principal reservoirs of Brucellae are small rodents.
c. Brucellae are obligate intracellular parasites that are usually identified by growth in human cell culture.
d. Brucellae infect reticuloendothelial cells in the liver, spleen, and bone marrow. ✔
1797. Which one of the following illnesses is NOT a zoonosis?
a. Q fever
b. Rocky Mountain spotted fever
c. Tularemia
d. Typhoid fever ✔
1798. Mosquitoes is/are the vector in the following disorder(s)
a. Onchocerciasis
b. Bancroftian filariasis ✔
c. African trypanosomiasis
d. Visceral leishmaniasis
1799. Sixteen residents in a retirement home have fever, malaise, and anorexia. These residents have taken their
meals prepared by the same kitchen. Blood cultures from of these residents grow Salmonella enterica
subsp. typhi. The primary reservoir of this organism is
a. dogs and cats
b. turkeys
c. people ✔
d. water
e. hen’s egg
1800. Each of the following statements concerning Rocky Mountain spotted fever is correct EXCEPT:
a. The disease is caused by a Rickettsia.
b. Headache, fever, and rash are characteristic features of the disease.
c. The causative organism forms beta-hemolytic colonies on blood agar. ✔
d. The disease occurs primarily east of the Mississippi.
1801. Each of the following statements concerning epidemic typhus is correct EXCEPT:
a. The disease is caused by a Rickettsia.
b. The Weil-Felix test can aid in diagnosis of the disease.
c. The causative organism is transmitted from rodents to humans by a tick. ✔
d. The disease is characterized by a rash.
1802. What organism is most likely responsible for bacterial endocarditis in men following urological
instrumentation.
a. Ureaplasma urealyticum
b. Enterococcus faecalis ✔
c. Streptococcus pyogenes
d. Pseudomonas aeruginosa
e. Viridans streptococci
1803. A 12-year-old boy from North Carolina presents to the emergency department with rash, fever, and severe
headache that began 3 days ago. The rash began on his arms and legs and then spread to the trunk. The
pediatrician notes conjunctival redness, and lab tests reveal proteinuria. Which of the following events
likely led to the child’s illness?
a. Cutting himself while butchering rabbits
b. Hiking in the woods ✔
c. Kissing
d. Not washing his hands
e. Eating undercooked meat
1804. After sporozoite gain entrance to human body it undergoes developmental cycle first in liver than in RBC,
only after which fever is seen. This incubation period varies between plasmodium species, and …………..
species has longest incubation period.
a. P. malariae ✔
b. P. ovale
c. P. falciparum
d. P. vivax
1805. Which of the following statement(s) regarding Plasmodium falciparum are true?
a. is the only cause of cerebral malaria
b. is associated with recurrent relapses after initial treatment because of liver hypnozoites
c. is the only malarial parasite causing greater than 20% parasitaemia ✔
d. causes benign tertian malaria
1806. Each of the following parasites is transmitted by mosquitoes EXCEPT:
a. Leishmania donovani ✔
b. Plasmodium vivax
c. Wuchereria bancrofti
d. Plasmodium falciparum
1807. A 35-year-old man comes to the emergency department complaining of high fever, chills, severe headache,
and confusion. He has recently returned from Africa. A peripheral blood smear reveals multiple ring
structures and crescent-shaped gametes. Which of the following organisms is the most likely cause?
a. Leishmania species
b. Plasmodium vivax
c. Plasmodium ovale
d. Plasmodium falciparum ✔
e. Plasmodium malariae
1808. Ticks are vectors for the transmission of each of the following diseases EXCEPT:
a. Tularemia
b. Lyme disease
c. Epidemic typhus ✔
d. Rocky Mountain spotted fever
1809. Each of the following statements concerning Q fever is correct EXCEPT:
a. Rash is a prominent feature. ✔
b. Farm animals are an important reservoir.
c. It is transmitted by respiratory aerosol.
d. It is caused by Coxiella burnetii.
1810. Each of the following statements concerning sleeping sickness is correct EXCEPT:
a. Sleeping sickness is caused by a trypanosome.
b. Sleeping sickness occurs primarily in tropical Africa.
c. Sleeping sickness can be diagnosed by finding eggs in the stool. ✔
d. Sleeping sickness is transmitted by tsetse flies.
1811. Trophozoites, Schizonts and gametocytes of all the malarial parasites are seen in the peripheral blood
smear except ;
a. P. falciparum ✔
b. P. malariae
c. P. vivax
d. P. ovale
1812. Which one of the following organisms principally infects vascular endothelial cells?
a. Rickettsia rickettsii ✔
b. Haemophilus influenzae
c. Salmonella typhi
d. Coxiella burnetii
1813. Regarding the effect of benzylpenicillin (penicillin G) on bacteria, which one of the following organisms is
LEAST likely to be resistant?
a. Enterococcus faecalis
b. Staphylococcus aureus
c. Streptococcus pyogenes ✔
d. Neisseria gonorrhoeae
1814. Each of the following statements concerning penicillin is correct EXCEPT:
a. Penicillin inhibits transpeptidases, which are required for cross-linking peptidoglycan.
b. The structure of penicillin resembles that of a dipeptide of alanine, which is a component of
peptidoglycan.
c. Penicillin is a bacteriostatic drug because autolytic enzymes are not activated. 11
d. An intact ß-lactam ring of penicillin is required for its activity.
1815. Each of the following statements concerning the mechanism of action of antimicrobial drugs is correct
EXCEPT:
a. Vancomycin acts by inhibiting peptidoglycan synthesis.
b. Quinolones, such as ciprofloxacin, act by inhibiting the DNA gyrase of bacteria.
c. Aminoglycosides such as streptomycin are bactericidal drugs that inhibit protein synthesis.
d. Erythromycin is a bactericidal drug that disrupts cell membranes by a detergent-like action. ✔
1816. Each of the following statements concerning the resistance of bacteria to antimicrobial drugs is correct
EXCEPT:
a. Resistance to chloramphenicol is known to be due to an enzyme that acetylates the drug.
b. Resistance to penicillin is known to be due to cleavage by ß-lactamase.
c. Resistance to tetracycline is known to be due to an enzyme that hydrolyzes the ester linkage. 11
d. Resistance to penicillin is known to be due to reduced affinity of transpeptidases.
1817. The use of antibiotics that inhibit or inactivate cellular ribosomes will result directly in the loss of which of
the following functions:
a. ATP production
b. Cell division
c. Protein synthesis ✔
d. DNA replication
1818. Each of the following statements concerning the mechanism of action of antibacterial drugs is correct
EXCEPT:
a. Cephalosporins are bactericidal drugs that inhibit the transpeptidase reaction and prevent cell wall
synthesis.
b. Erythromycin is a bacteriostatic drug that inhibits protein synthesis by blocking translocation of the
polypeptide.
c. Aminoglycosides are bacteriostatic drugs that inhibit protein synthesis by activating ribonuclease, which
degrades mRNA. ✔
d. Tetracyclines are bacteriostatic drugs that inhibit protein synthesis by blocking tRNA binding.
1819. Each of the following statements concerning the mechanisms of resistance to antimicrobial drugs is correct
EXCEPT:
a. Resistance to some drugs is due to a chromosomal mutation that alters the receptor for the drug.
b. R factors are plasmids that carry the genes for enzymes that modify one or more drugs.
c. Resistance to some drugs is due to transposon genes that code for enzymes that inactivate the drugs.
d. Resistance genes are rarely transferred by conjugation. ✔
1820. Regarding viroids, which one of the following statements is the MOST accurate?
a. They cause tumors in experimental animals.
b. They consist of RNA without a protein or lipoprotein outer coat. ✔
c. They are defective viruses that are missing the DNA coding for the matrix protein.
d. They require an RNA polymerase in the particle for replication to occur.
1821. Reasonable mechanisms for viral persistence in infected individuals include all of the following EXCEPT:
a. Generation of defective-interfering particles
b. Virus-mediated inhibition of host DNA synthesis ✔
c. Host tolerance to viral antigens
d. Integration of a provirus into the genome of the host
1822. A 5-year-old presents to the pediatrician with complaints of a sore throat. Her mother also noticed that
both of her eyes were slightly red. Examination reveals rhinopharyngitis with bilateral conjunctivitis. What
activity likely led to the above illness?
a. Eating undercooked shellfish
b. Hiking in a heavily wooded area
c. Traveling to a developing country
d. Playing with toys in a day care center
e. Swimming in a community pool ✔
1823. Regarding the serologic diagnosis of infectious mononucleosis, which one of the following is CORRECT?
a. A heterophil antibody is formed that reacts with a capsid protein of Epstein–Barr virus.
b. A heterophil antigen occurs following infection with cytomegalovirus.
c. A heterophil antibody is formed that agglutinates sheep or horse red blood cells. ✔
d. A heterophil antigen occurs that cross-reacts with Proteus OX19 strains.
1824. What is the primary means of spread for measles?
a. Animal bite
b. Respiratory droplet spread ✔
c. Fomite spread
d. Tick bite
e. Fecal-oral
1825. An 11 -month-old infant was brought to the emergency department with difficulty breathing and wheezing.
History and physical examination reveal a slight fever, cough, and rhinorrhea that began about 2 days
before. Analysis of the sputum reveals normal flora with the presence of giant multinucleated cells. Which
of the following is the most likely cause?
a. Measles
b. B 19
c. Respiratory syncytial virus ✔
d. Parainfluenza
e. Influenza
1826. Each of the following statements concerning retroviruses is correct EXCEPT:
a. During infection, the virus synthesizes a DNA copy of its RNA, and this DNA becomes covalently
integrated into host cell DNA.
b. The virion is enveloped and enters cells via an interaction with specific receptors on the host cell.
c. The viral genome consists of three segments of double-stranded RNA. ✔
d. The virion carries an RNA-directed DNA polymerase encoded by the viral genome.
1827. Viruses enter cells by adsorbing to specific sites on the outer membrane of cells. Each of the following
statements regarding this event is correct EXCEPT:
a. If the sites are occupied, interference with virus infection occurs.
b. The interaction can be prevented by neutralizing antibody.
c. The interaction determines the specific target organs for infection.
d. The interaction determines whether the purified genome of a virus is infectious. ✔
1828. Which one of the following statements about virion structure and assembly is CORRECT?
a. The symmetry of virus particles prevents inclusion of any nonstructural proteins, such as enzymes.
b. Helical nucleocapsids are found primarily in DNA viruses.
c. Most viruses acquire surface glycoproteins by budding through the nuclear membrane.
d. Enveloped viruses use a matrix protein to mediate interactions between viral glycoproteins in the plasma
membrane and structural proteins in the nucleocapsid. ✔
1829. Each of the following statements concerning viruses is correct EXCEPT:
a. The proteins on the surface of the virus mediate the entry of the virus into host cells.
b. Viruses replicate by binary fission. ✔
c. Neutralizing antibody is directed against proteins on the surface of the virus.
d. Viruses can reproduce only within cells.
1830. In the U.S., a baby has the greatest chance of acquiring which virus in utero?
a. Respiratory syncytial virus
b. Rubella virus
c. Hepatitis B virus
d. Cytomegalovirus ✔
e. Herpes simplex virus
1831. Each of the following statements about both measles virus and rubella virus is correct EXCEPT:
a. They are transmitted by respiratory aerosol.
b. Their virions contain an RNA polymerase. ✔
c. They have a single antigenic type.
d. They are RNA enveloped viruses.
1832. A 5-year-old girl presents with a fever and a generalized macular rash that is most dense on the scalp and
trunk of the body. Several waves of lesions appear, one after another, and evolve rapidly into vesicles and
then pustules over several days. The most likely disease and causal agent is
a. Whitlow infection due to herpes simplex virus type 1
b. Infectious mononucleosis due to the Epstein-Barr virus
c. Exanthem subitum due to cytomegalovirus
d. Chickenpox due to the varicella-zoster virus ✔
e. Herpetic gingivostomatitis due to the varicella-zoster virus
1833. Which of the following statements concerning antigenic drift in influenza viruses is correct?
a. It results in major antigenic changes
b. It is exhibited by only influenza A viruses
c. It is due to frame shift mutations in viral genes
d. It affects predominantly the matrix protein
e. It results in new sub types over time ✔
1834. Which one of the following viruses possesses a genome of single-stranded RNA that is infectious when
purified?
a. Influenza virus
b. Measles virus
c. Rotavirus
d. Poliovirus ✔
1835. Which one of the following statements about genital herpes is LEAST accurate?
a. Acyclovir reduces the number of recurrent disease episodes by eradicating latently infected cells. ✔
b. Initial disease episodes are generally more severe than recurrent episodes.
c. Multinucleated giant cells with intranuclear inclusions are found in the lesions.
d. Genital herpes can be transmitted in the absence of apparent lesions.
1836. Each of the following statements concerning the antigenicity of influenza A virus is correct EXCEPT:
a. The protein involved in antigenic drift is primarily the internal ribonucleoprotein. ✔
b. Antigenic shifts, which represent major changes in antigenicity, occur infrequently and are due to the
reassortment of segments of the viral genome.
c. Antigenic shifts affect both the hemagglutinin and the neuraminidase.
d. The worldwide epidemics caused by influenza A virus are due to antigenic shifts.
e. Many viruses mature by budding through the outer membrane of the host cell.
1837. Each of the following statements regarding these viruses is correct EXCEPT:
a. Some new viral antigens appear on the surface of the host cell.
b. Some of these viruses contain host cell lipids.
c. Some of these viruses cause multinucleated giant cell formation.
d. Some of these viruses do not have an envelope. ✔
1838. Each of the following viruses possesses double-stranded nucleic acid as its genome EXCEPT:
a. Adenovirus
b. Coxsackie virus ✔
c. Rotavirus
d. Herpes simplex virus
1839. Viruses are obligate intracellular parasites. Each of the following statements concerning this fact is correct
EXCEPT:
a. Viruses cannot synthesize proteins outside of cells.
b. Viruses cannot generate energy outside of cells.
c. Viruses must degrade host cell DNA in order to obtain nucleotides. ✔
d. Enveloped viruses require host cell membranes to obtain their envelopes.
1840. Each of the following statements concerning Coccidioides immitis is correct EXCEPT:
a. In the body, spherules containing endospores are formed.
b. A rising titer of complement-fixing antibody indicates disseminated disease.
c. The mycelial phase of the organism grows primarily in the soil, which is its natural habitat.
d. Most infections are symptomatic and require treatment with amphotericin B. ✔
1841. Each of the following statements concerning Histoplasma capsulatum is correct EXCEPT:
a. The natural habitat of H. capsulatum is the soil, where it grows as a mold.
b. Within the body, H. capsulatum grows primarily intracellularly within macrophages.
c. Passive immunity in the form of high titer antibodies should be given to those known to be exposed. ✔
d. H. capsulatum is transmitted by airborne conidia, and its initial site of infection is the lung.
1842. An obese 32-year-old diabetic woman presents with complaint of red and painful skin in her abdominal
skin folds. Examination reveals a creamy white material at the base of the fold. It is erythematous
underneath and extends beyond the creamy material. Microscopic examination of the exudate reveals oval
budding structures (3 X 6 µm) mixed with more budding elongated forms. The most likely causal agent is
a. Epidermophyton floccosum
b. Microsporum canis
c. Aspergillus fumigatus
d. Candida albicans ✔
e. Sporothrix schenckii
1843. A 33-year-old HIV-positive man complains of headache and blurred vision. Physical examination reveals
papilledema and ataxia. A head CT scan is normal, but CSF obtained by lumbar puncture reveals
encapsulated organisms visible by India ink. Which of the following is true concerning this organism?
a. It is an encapsulated non dimorphic yeast found worldwide ✔
b. It exists as a mycelial form at room temperature and a yeast at 37°C
c. It can also be seen as “spaghetti and meatballs” on KOH stain
d. It is a nonencapsulated dimorphic yeast that reproduces by budding
e. It consists of branching septate hyphae
1844. Each of the following statements concerning fungi and protozoa is correct EXCEPT:
a. Both fungi and protozoa generate energy in mitochondria.
b. Fungi possess a cell wall, whereas protozoa do not.
c. Both fungi and protozoa use flagella as their organ of motility. ✔
d. Both fungi and protozoa are eukaryotic organisms.
1845. Fungal cells that reproduce by budding are seen in the infected tissues of patients with:
a. Mycetoma, candidiasis, and mucormycosis
b. Sporotrichosis, mycetoma, and aspergillosis
c. Tinea corporis, tinea unguium, and tinea versicolor
d. Candidiasis, cryptococcosis, and sporotrichosis ✔
1846. An 18-year-old high school student in rural north Mississippi develops fever, cough, and chest pain. The
cough, associated with weight loss, persisted. Because of poor performance at football practice, he was
advised to see a physician. Lymph node biopsies stained with H and E reveal granulomatous inflammation
and macrophages engorged with oval structures measuring 2-4 µm. Cultures incubated at room
temperature grow powdery white colonies, which on microscopic study have tuberculate spores. The high
school student most likely acquired the infection from which of the following?
a. Contaminated drinking water
b. Cat feces
c. Soil enriched with bird excrement ✔
d. Another human via respiratory secretions
e. Desert sand
1847. Each of the following statements concerning Blastomyces dermatitidis is correct EXCEPT:
a. B. dermatitidis causes granulomatous skin lesions.
b. B. dermatitidis infection is commonly diagnosed by serologic tests because it does not grow in culture. ✔
c. B. dermatitidis grows as a mold in the soil in North America.
d. B. dermatitidis is a dimorphic fungus that forms yeast cells in tissue.
1848. A 42-year-old man who has AIDS presents to his physician with progressively increasing dyspnea over the
past 3 weeks. He also complains of a dry, painful cough, fatigue, and low-grade fever. A chest x-ray reveals
bilateral symmetrical interstitial and alveolar infiltration. Which of the following agents is the most likely
cause of the above?
a. Histoplasma capsulatum
b. Cryptococcus neoformans
c. Pneumocystis jiroveci ✔
d. Cryptosporidium parvum
e. Toxoplasma gondii
1849. Fungi often colonize lesions due to other causes. Which one of the following is LEAST likely to be present
as a colonizer?
a. Aspergillus
b. Sporothrix ✔
c. Candida
d. Mucor
1850. When immune complexes from the serum are deposited on glomerular basement membrane, damage to the
membrane is caused mainly by:
a. Gamma interferon
b. Enzymes released by polymorphonuclear cells ✔
c. Phagocytosis
d. Cytotoxic T cells
1851. Toxoplasma gondii is an intracellular parasite that lives inside phagocytic and nonphagocytic cells by
generating its own intracellular vesicle. This may allow it to avoid recognition and killing by CDS+
lymphocytes, which require the presentation of foreign peptides transported into the endoplasmic
reticulum and loaded onto MHC molecules that have
a. a beta2 domain instead of a beta 2 microglobulin
b. a single transmembrane domain ✔
c. invariant chains
d. two similar chains
e. a peptide-binding groove
1852. Each of the following statements concerning class II MHC proteins is correct EXCEPT:
a. They are involved in the presentation of antigen by macrophages.
b. They are found on the surface of both B and T cells. ✔
c. They have a high degree of polymorphism.
d. They have a binding site for CD4 proteins.
1853. Macrophages are major source of:
a. IL-5
b. IFN?
c. IL-1 ✔
d. IL-7
1854. A patient skin-tested with purified protein derivative (PPD) to determine previous exposure to
Mycobacterium tuberculosis develops induration at the skin test site 48 hours later. Histologically, the
reaction site would MOST probably show:
a. Eosinophils
b. Neutrophils
c. Helper T cells and macrophages ✔
d. B cells
1855. Each of the following statements concerning immunologic tolerance is correct EXCEPT:
a. Tolerance is more easily induced in T cells than in B cells.
b. Tolerance is not antigen-specific (i.e., paralysis of the immune cellsresults in a failure to produce a
response against many antigens). ✔
c. Tolerance is more easily induced by simple molecules than by complex ones.
d. Tolerance is more easily induced in neonates than in adults.
1856. Which one of the following substances is NOT released by activated helper T cells?
a. Interleukin-2
b. Interleukin-4
c. Gamma interferon
d. Alpha interferon ✔
1857. Cytotoxic T cells induced by infection with virus A will kill target cells:
a. Infected by virus A and identical at class I MHC loci of the cytotoxic T cells ✔
b. From the same host infected with any virus
c. Infected with a different virus and identical at class II MHC loci of the cytotoxic cells
d. Infected with a different virus and identical at class I MHC loci of the cytotoxic cells
e. Infected by virus A and identical at class II MHC loci of the cytotoxic T cells
1858. Antigen-presenting cells that activate helper T cells must express which one of the following on their
surfaces?
a. Class I MHC antigens
b. Class II MHC antigens ✔
c. Gamma interferon
d. IgE
1859. Your patient is a child who has no detectable T or B cells. This immunodeficiency is most probably the
result of a defect in
a. The membrane attack complex of complement
b. T cell–B cell interaction
c. Stem cells originating in the bone marrow ✔
d. The thymus
1860. Which category of hypersensitivity BEST describes hemolytic disease of the newborn caused by Rh
incompatibility ?
a. Immune complex
b. Atopic or anaphylactic
c. Delayed
d. Cytotoxic ✔
1861. Wheal and flare is which type of hypersensitivity reaction?
a. Type V
b. Type IV
c. Type I ✔
d. Type II
1862. Mediated through allergen specific IgE A patient with severe asthma gets no relief from antihistamines.
The symptoms are MOST likely to be caused by:
a. Interleukin-2
b. Bradykinin
c. Serotonin
d. Slow-reacting substance A (leukotrienes) ✔
1863. A patient with a central nervous system disorder is maintained on the drug methyldopa. Hemolytic anemia
develops, which resolves shortly after the drug is withdrawn. This is MOST probably an example of:
a. Atopic hypersensitivity
b. Immune-complex hypersensitivity
c. Cell-mediated hypersensitivity
d. Cytotoxic hypersensitivity ✔
1864. Type I hypersensitivity includes all of the following except:
a. Hay fever
b. Extrinsic asthma
c. Anaphylaxis
d. Autoimmune hemolytic anemia ✔
1865. Bone marrow transplantation in immunocompromised patients presents which major problem?
a. Potentially lethal graft-versus-host disease ✔
b. Inability to use a live donor
c. High risk of T-cell leukemia
d. Delayed hypersensitivity
1866. A 40-year-old woman has a history of chronic inflammation of the small joints of the hands bilaterally. You
suspect rheumatoid arthritis. Which one of the following statements is the MOST accurate regarding the
pathogenesis of this disease?
a. It is caused by antibody against human IgG-forming immune complexes within the joints.
b. It is caused by superantigens inducing the release of large amounts of lymphokines from helper T cells
within the joints. ✔
c. It is caused by the release of mediators from mast cells when environmental agents cross-link adjacent
IgEs within the joints.
d. It is caused by sensitized CD4-positive T lymphocytes and macrophages invading the joints.
1867. Type I hypersensitivity is mediated by which of the following immunoglobulins?
a. IgG
b. IgM
c. IgA
d. IgE ✔
1868. The principal difference between cytotoxic (type II) and immune complex (type III) hypersensitivity is:
a. The class (isotype) of antibody
b. Whether the antibody reacts with the antigen on the cell or reacts with antigen before it interacts with the
cell ✔
c. The participation of complement
d. The participation of T cells
1869. A child stung by a bee experiences respiratory distress within minutes and lapses into unconsciousness.
This reaction is probably mediated by:
a. IgG antibody
b. IgM antibody
c. Sensitized T cells
d. Complement
e. IgE antibody ✔
1870. A 4-year-old boy presents to his pediatrician with intense perianal itching. His mother explains that the
child has also been extremely irritable during the day and has not been sleeping well at night. Eggs with a
flattened side were identified by the laboratory technician from a piece of scotch tape brought in by the
parent. Infection with which of the following organisms is most likely?
a. Ascaris lumbricoides
b. Entamoeba histolytica
c. Echinococcus granulosus
d. Enterobius vermicularis ✔
e. Trichuris trichiura
1871. Each of the following statements concerning sleeping sickness is correct EXCEPT:
a. Sleeping sickness is caused by a trypanosome.
b. Sleeping sickness occurs primarily in tropical Africa.
c. Sleeping sickness can be diagnosed by finding eggs in the stool. ✔
d. Sleeping sickness is transmitted by tsetse flies.
1872. An HIV-positive patient with a CD4+ count of 47 presents with diarrhea. Acidfast structures are found in
the stool. From this finding, which of the following is true?
a. Infection is short lasting and self-resolving and requires no treatment
b. Even with the best treatment, the infection may be unrelenting ✔
c. If treated with antibiotics, the infection should resolve in 3-6 days
d. Infection will resolve only with a combination of antituberculous drugs, and then it may take weeks
e. Infection could have been prevented by avoiding cat feces and undercooked or raw meat
1873. Each of the following statements concerning Toxoplasma gondii is correct EXCEPT:
a. T. gondii can be transmitted across the placenta to the fetus.
b. T. gondii can cause encephalitis in immunocompromised patients.
c. T. gondii can be transmitted by cat feces.
d. T. gondii can be diagnosed by finding trophozoites in the stool. ✔
1874. A 35-year-old captain in the army reserves has been plagued by a painful, erosive lesion near his ear lobe
since his return from Operation Desert Storm several years ago. He denies exposure to the toxic by-
products of burning oil fields. Punch biopsy of the leading edge of the erosion reveals macrophages
distended with oval amastigotes. How was this infection acquired?
a. Bite of Anopheles mosquito
b. Bite of tsetse fly
c. Bite of reduviid bug
d. Fecal contamination of food
e. Contact with contaminated drinking water
f. Bite of sandfly ✔
1875. Each of the following statements concerning Ascaris lumbricoides is correct EXCEPT:
a. A. lumbricoides is one of the largest nematodes.
b. Both dogs and cats are intermediate hosts of A. lumbricoides. ✔
c. A. lumbricoides is transmitted by ingestion of eggs.
d. A. lumbricoides can cause pneumonia.
1876. Each of the following statements concerning Trichomonas vaginalis is correct EXCEPT:
a. T. vaginalis is transmitted sexually.
b. T. vaginalis causes bloody diarrhea. ✔
c. T. vaginalis can be diagnosed by visualizing the trophozoite.
d. T. vaginalis can be treated effectively with metronidazole.
1877. Each of the following statements concerning hookworm infection is correct EXCEPT:
a. Hookworm infection can be diagnosed by finding the trophozoite in the stool. ✔
b. Hookworm infection is caused by Necator americanus.
c. Hookworm infection is acquired by humans when filariform larvae penetrate the skin.
d. Hookworm infection can cause anemia.
1878. Each of the following statements concerning kala-azar is correct EXCEPT:
a. Kala-azar is transmitted by the bite of sandflies.
b. Kala-azar can be diagnosed by finding amastigotes in bone marrow.
c. Kala-azar occurs primarily in rural Latin America. ✔
d. Kala-azar is caused by Leishmania donovani.
1879. Mature cyst of Entamoeba histolytica is:
a. binucleate
b. quadrinucleate ✔
c. uninucleate
d. octanucleate
1880. The mother of a 4-year-old child notes that her child is sleeping poorly and scratching his anal area. You
suspect the child may have pinworms. Which one of the following is the BEST method to make that
diagnosis?
a. Examine the stool for the presence of trophozoites
b. Examine transparent adhesive tape for the presence of eggs ✔
c. Examine the stool for the presence of cysts
d. etermine the titer of IgE antibody against the organism
e. Examine a blood smear for the presence of microfilaria
1881. Infective stage of Entamoeba histolytica is:
a. trophozoite
b. quadrinucleate cyst ✔
c. sporozoite
d. filariform larvae
1882. Each of the following statements concerning Schistosoma haematobium is correct EXCEPT:
a. Snails are intermediate hosts of S. haematobium.
b. S. haematobium infection predisposes to bladder carcinoma.
c. S. haematobium eggs have no spine. ✔
d. S. haematobium is acquired by humans when cercariae penetrate the skin.
1883. Each of the following parasites is transmitted by mosquitoes EXCEPT:
a. Wuchereria bancrofti
b. Plasmodium falciparum
c. Plasmodium vivax
d. Leishmania donovani ✔
1884. At a school nurse’s request, a clinic in rural South Carolina sees a 9- year-old girl who appears listless and
inattentive, although hearing and visual testing has been within normal limits. The physician finds the child
thin, with the “potbelly” of malnutrition, and orders a fecal exam and CBC. The CBC reveals a microcytic,
hypochrornic anemia, and the fecal exam detects brown, oval nematode eggs approximately 65 microns in
size, too numerous to count. What was the most likely means by which this child was infected?
a. Ingestion of ova
b. Mosquito transmission of sporozoites
c. Skin penetration by larvae ✔
d. Ingestion of larvae
e. Ingestion of cysts in muscle
1885. Each of the following statements concerning hydatid cyst disease is correct EXCEPT:
a. The disease occurs primarily in tropical Africa. ✔
b. The disease is caused by Echinococcus granulosus.
c. The disease is caused by a parasite whose adult form lives in dogs’intestines.
d. The cysts occur primarily in the liver.
1886. Each of the following statements concerning Diphyllobothrium latum is correct EXCEPT:
a. D. latum has operculated eggs.
b. D. latum is transmitted by undercooked fish.
c. D. latum is a tapeworm that has a scolex with a circle of hooks. ✔
d. D. latum causes a megaloblastic anemia due to vitamin B12 deficiency.
1887. A 24-year-old primiparous woman in her eighth month of gestation develops a positive IgM titer to
Toxoplasma gondii for the first time. She should be advised by her physician that
a. retinochoroiditis can be prevented by drug treatment of an infant with a positive IgM response ✔
b. future infections can be avoided by proper vaccination and worming of cats
c. a newborn with a positive anti-Toxoplasma IgG response should be treated with anti-parasitics
d. major organ damage can be reversed by prompt treatment of the newborn this child and all future fetuses
are likely to be infected
1888. Each of the following parasites passes through the lung during human infection EXCEPT:
a. Wuchereria bancrofti ✔
b. Ascaris lumbricoides
c. Necator americanus
d. Strongyloides stercoralis
1889. Each of the following parasites is transmitted by eating inadequately cooked fish or seafood EXCEPT:
a. Clonorchis sinensis
b. Paragonimus westermani
c. Ancylostoma duodenale ✔
d. Diphyllobothrium latum
1890. Laboratory diagnosis of a patient with a suspected liver abscess due to Entamoeba histolytica should
include:
a. Stool examination and indirect hemagglutination test ✔
b. Indirect hemagglutination test and skin test
c. Xenodiagnosis and string test
d. Stool examination and blood smear
1891. Children at day care centers in the United States have a high rate of infection with which one of the
following?
a. Necator americanus
b. Enterobius vermicularis ✔
c. Entamoeba histolytica
d. Ascaris lumbricoides
1892. Mosquitoes is/are the vector in the following disorder(s)
a. Onchocerciasis
b. Bancroftian filariasis ✔
c. African trypanosomiasis
d. Visceral leishmaniasis
1893. The main anatomic location of Schistosoma mansoni adult worms is:
a. Lung alveoli
b. Renal tubules
c. Bone marrow
d. Intestinal venules ✔
1894. A 13-year-old boy from India was brought to the emergency room with a prolapsed rectum. Examination
of the rectum reveals small worms that resemble whips attached to the mucosa. A stool sample reveals eggs
that are barrel shaped, with bipolar plugs. Which of the following is the most likely cause?
a. Echinococcus granulosus
b. Entamoeba histolytica
c. Enterobius vermicularis
d. Trichuris trichiura ✔
e. Ascaris lumbricoides
f. Giardia lamblia
1895. Which one of the following statements concerning immunoglobulin allotypes is CORRECT?
a. Allotypes are confined to the variable regions.
b. Allotypes are found only on heavy chains.
c. Allotypes are due to genetic polymorphism within a species. ✔
d. Allotypes are determined by class I MHC genes.
1896. The MOST important protective function of the antibody stimulated by tetanus immunization is:
a. To opsonize the pathogen (Clostridium tetani)
b. To prevent growth of the pathogen
c. To neutralize the toxin of the pathogen ✔
d. To prevent adherence of the pathogen
1897. Inactivation of a toxin by an antibody is termed:
a. Opsonization
b. Neutralization ✔
c. Lyophilization
d. Lysis
e. Attenuation
1898. An epitope is
a. the antigen determinant site ✔
b. a B-cell
c. an antibody
d. a hapten
1899. Which of the following antibodies have a positive effect on babies through breast feeding?
a. IgG
b. IgA ✔
c. IgM
d. IgD
e. IgE
1900. Isotypes of Antibodies refer to variations in the:
a. light chain variable region
b. heavy chain variable region ✔
c. heavy chain constant region
d. light chain constant region
1901. DNA vaccine contain………….. DNA that stimulates cells to make………….antigens.
a. Human/ RNA
b. Microbial/ Carbohydrate
c. Human/Lipid
d. Microbial/Protein ✔
e. Human/ Protein
1902. Regarding the primary and secondary antibody responses, which one of the following statements is MOST
accurate?
a. The amount of IgG made in the secondary response is greater than the amount made in the primary
response. ✔
b. The lag phase is shorter in the primary response than in the secondary response.
c. Antigen must be processed and presented in the primary response but not in the secondary response.
d. In the primary response, memory B cells are produced, but memory T cells are not.
e. The IgM made in the primary response is made primarily by memory B cells.
1903. “Isotype switching” of immunoglobulin classes by B cells involves:
a. Simultaneous insertion of VH genes adjacent to each CH gene
b. Activation of homologous genes on chromosome 6
c. Successive insertion of a VH gene adjacent to different CH genes ✔
d. Switching of light chain types (kappa and lambda)
1904. Of the following choices, the MOST important function of antibody in host defenses against bacteria is:
a. Inhibition of bacterial protein synthesis
b. Activation of lysozyme that degrades the cell wall
c. Acceleration of proteolysis of exotoxins
d. Facilitation of phagocytosis ✔
1905. In agglutination reactions, the antigen is a ……………… in precipitation reactions, the antigen is a
……………..
a. Protein/ Carbohydrate
b. Soluble molecule/ Whole cell
c. Bacterium/ Virus
d. Whole cell/ Soluble molecule ✔
e. Protein/ Antibody
1906. Each of the following statements concerning haptens is correct EXCEPT:
a. A hapten can combine with (bind to) an antibody.
b. Haptens must be processed by CD8+ cells to become immunogenic. ✔
c. A hapten cannot induce an antibody by itself; rather, it must be bound to a carrier protein to be able to
induce antibody.
d. In both penicillin-induced anaphylaxis and poison ivy, the allergens are haptens.
1907. An antibody directed against the idiotypic determinants of a human IgG antibody would react with:
a. The Fc part of the IgG
b. All human kappa chains
c. All human gamma chains
d. An IgM antibody produced by the same plasma cell that produced the IgG ✔
1908. An Rh-negative woman married to a heterozygous Rh-positive man has three children. The probability
that all three of their children are Rh-positive is:
a. 1:2
b. 1:8 ✔
c. 1:4
d. Zero
1909. An immunoglobulin is a
a. glycoprotein ✔
b. fatty acid
c. protein
d. carbohydrate
1910. An antigen that overstimualtes the immune system by binding nonspecifically to MHC on antigen presenting cells
is termed
a. Non specific antigen
b. Toxic shock syndrome
c. Super necrotic
d. Super antigen ✔
e. Epitope
1911. Individuals of blood group type AB:
a. Are Rh(D)-negative
b. Have circulating anti-A and anti-B antibodies
c. Are “universal recipients” of transfusions ✔
d. Have the same haplotype
1912. The membrane IgM and IgD on the surface of an individual B cell:
a. Are identical except for their CH regions ✔
b. Have identical heavy chains but different light chains
c. Are identical except for their VH regions
d. Have different VH and VL regions
1913. Which of the following antibodies would most likely be found in body secretions such as tears, milk, saliva and
mucus?
a. IgG
b. IgE
c. IgM
d. IgA ✔
e. IgD
1914. Which one of the following is NOT true of the gene segments that combine to make up a heavy chain gene?
a. A V segment and a J segment are preselected by an antigen to make up the
b. variable-region portion of the gene. ✔
c. Many V region segments are available.
d. Several J segments and several D segments are available.
e. V, D, and J segments combine to encode the antigen-binding site.
1915. Mitochondria are missing in
a. Filamentous fungi
b. Cestodes
c. Protozoan parasites
d. Yeasts
e. Bacteria ✔
1916. Which one of the following statements is the MOST accurate comparison of human, bacterial, and fungal cells?
a. Human and bacterial cells have plasmids, whereas fungal cells do not
b. Human and fungal cells have a similar cell wall, in contrast to bacteria,
c. whose cell wall contain peptidoglycan
d. Human and fungal cells have similar robosomes, whereas bacterial ribosomes are different ✔
e. Human cell undergo mitosis, whereas neither bacteria nor fungi do
1917. Each of the following statements concerning bacterial spores is correct EXCEPT:
a. They are formed by gram-positive rods
b. They can be killed by being heated to 121oC for 15 minutes
c. Their survival ability is based on their enhanced metabolic activity ✔
d. They contain much less water than bacterial cells
1918. Which of the following BEST describes the mode of action of endotoxin?
a. Degrades lecithin in cell membranes
b. Blocks release of acetylcholine
c. Causes the release of tumor necrosis factor ✔
d. Inactivates elongation factor 2
1919. A cancer chemotherapy patient has to have her intravenous port revised after it becomes blocked and the catheter
is found to contain bacterial contaminants. Which of the following attributes is most likely to be a factor in this
pathogenesis?
a. Possession of IgA protease
b. Possession of pili
c. Peptidoglycan layer
d. Ergosterol containing membrane
e. Biofilm production ✔
1920. Each of the following statements concerning bacterial spores is correct EXCEPT:
a. Spores are resistant to boiling
b. Spores are formed primarily by organisms of the genus Neisseria ✔
c. Spores are metabolically inactive and contain dipicolinic acid, a calcium chelator
d. Spores are formed under adverse environmental conditions such as the absence of a carbon source
1921. Usually, bacteria form more endospores in response to:
a. Nutrient surplus
b. Adverse environmental stress ✔
c. Colony formation
d. Need for reproduction
1922. Each of the following statements concerning endotoxin is correct EXCEPT:
a. They are part of the bacterial cells wall, whereas exotoxins are not
b. They are less potent (ie, less active on a weight basis) than exotoxins
c. They are more stable on heating than exotoxins
d. They bind to specific cells receptors whereas exotoxins do not ✔
1923. Each of the following statements regarding the selective action of antibiotics on bacteria is correct EXCEPT:
a. Penicillins affect bacteria rather than human cells because bacteria have a cell wall, whereas human cells
do not.
b. Sulfonamides affect folic acid synthesis in bacteria, a pathway that does not occur in human cells. ✔
c. Isoniazid affects the DNA polymerase of bacteria but not that of human cells.
d. Chloramphenicol affects the large subunit of the bacterial ribosome, which is different from the large
subunit of the human ribosome.
1924. A 12-year-old boy presents to his pediatrician with fever, malaise, and a sore throat. Physical examination reveals
a fever of 103°F, cervical lymphadenopathy, and pharyngeal erythema. A swab is taken from some of the tonsillar
exudate and cultured on blood agar. Culture reveals beta hemolytic, gram-positive cocci, and a rapid antigen test
is positive. What is the major component that protects the causal agent from osmotic damage?
a. Lipopolysaccharide
b. Polysaccharide
c. Phospholipids
d. Teichoic acid
e. Peptidoglycan ✔
1925. The effect of endotoxin include each of the following EXCEPT:
a. Activation of the coagulation cascade
b. Fever
c. Hypotension
d. Opsonization
1926. Bacterial surface structures that show antigenic diversity include each of the following EXCEPT:
a. Peptidoglycan ✔
b. Flagella
c. Pili
d. Capsules
1927. Which of the following microscopic techniques provide three-dimensional images of a bacterial cell?
a. Transmission electron microscopy
b. Scanning electron microscopy ✔
c. Dark-field microscopy
d. Fluorescent microscopy
1928. Each of the following statements concerning the surface structures of bacteria is correct EXCEPT:
a. Pili mediate the interaction of bacteria with mucosal epithelium
b. Polysaccharide capsules retard phagocytosis
c. Both gram-negative rods and cocci have lipopolysaccharide “endotoxin” intheir cell wall
d. Bacterial flagella are nonantigenic in humans because they closely resemble human flagella in chemical
composition ✔
1929. Each of the following statements concerning exotoxins are correct EXCEPT:
a. When treated chemically, some exotoxins lose their toxicity and can be used as immunogens in vaccine
b. Some exotoxins are capable of causing disease in purified form, free of any bacteria
c. Some exotoxins act in the gastrointestinal tract to cause diarrhea
d. Some exotoxins contain lipopolysaccharides as the toxic components ✔
1930. Each of the following statements concerning the structure and chemical composition of bacteria is correct
EXCEPT:
a. Some gram-positive cocci contain a layer of teichoic acid external to the peptidoglycan
b. Some gram-negative rods contain lipid A in their cell wall
c. Some mycoplasmas contain pentaglycine in their peptidoglycan ✔
d. Some gram-positive rods contain dipicolinic acid in their spores
1931. The presence of a capsule around bacterial cells usually indicates their increased disease-causing potential and
resistance to disinfection. Capsules are generally viewed by:
a. Scanning electron microscopy
b. Ziehl-Neelsen staining
c. Gram staining
d. Negative staining ✔
1932. What structure is most responsible for triggering Gram-negative shock?
a. Capsule
b. Periplasmic space
c. Peptidoglycan- teichoic acid fragments
d. Outer Membrane ✔
e. Heat Shock Proteins
1933. What is an organism called that respires on the presence of oxygen and ferments in the absence of oxygen?
a. Facultative anaerobe ✔
b. Aerobe
c. Anaerobe
d. Facultative aerobe
e. Microaerophile
1934. The MAIN host defense against bacterial exotoxins is
a. Modulation of host cell receptors in response to the toxin
b. IgG and IgM antibodies ✔
c. Activated macrophages secreting proteases
d. Helper T cells